Свежие комментарии

Закон Луссера

Помнится, в Юном Технике за 1981 год была опубликована байка о том, как было изобретено дублирование магнето на заре авиации — ученик гимназии обратил внимание на то, что у людей два глаза, и, даже если подбить один глаз, второй продолжает видеть. С этим размышлением он пришел к известному авиатору Уточкину, который и наградил будущего конструктора двигателей десятью рублями.

Идея дублировать важные элементы сложных систем не нова и очевидна. Но вот когда это на самом деле стоит усилий, а когда нет можно определить только строгим математическим расчетом. Чтобы быть уверенным в том, что ваш самолет или любая другая сложная конструкция не откажет в самый неподходящий момент, существуют достаточно простые методики оценки общей надежности системы, доступные всем, кто не прогуливал школьную математику.

Итак, знакомьтесь: закон Луссера & co.


Two pilots
Вообще, тервер — штука совершенно не интуитивная. Интуиция подсказывает, что надежность всей цепочки определяется надежностью самого слабого звена. Так полагал даже всем известный фон Браун! Однако это весьма ошибочное мнение.

Но начнем с определения вероятности. В быту мы привыкли к процентам — что-то на 90% надежно, что-то на 50%. Для расчетов это не годится. Вероятность записывают числом от 0 до 1, где 1 соответствует полностью достоверному событию (которое в результате испытаний обязательно произойдет), а 0 — невозможному. Таким образом, вероятность в 99% (одна неудача случается в среднем на 100 попыток) мы запишем как 0.99, а вероятность в 50%, она же 1/2 (классический бросок монеты с равновероятным выпадением орла или решки) как 0.5.

Ну и самый главный закон тервера состоит в том, что вероятности совместных событий (т.е. тех, которые должны обязательно случиться,причем не обязательно в один и тот же момент времени, главное чтобы они все случились) перемножаются. Если с неба регулярно падают кирпичи и вероятность выжить за день равна 0.5 (т.е. 50%), то вероятность выжить в течении двух дней (надо выжить И сегодня И завтра, т.е. мы ставим требованием совместные события) равна произведению этих вероятностей: 0.5*0.5=0.25. Упс, вероятность прожить в таких условиях два дня всего лишь 25%. А шансы выжить в течении недели уже меньше процента (0.5^7=0.0078125). Если надеть каску и повысить выживаемость в каждом отдельном случае до 0.75, то шансы прожить неделю повысятся где-то до 0.7^7=0.08235423, т.е. примерно до 8 процентов.

Robert Lusser
Удивительно, но формула Луссера, прямо вытекающая из основ теории вероятностей, была сформулирована очень поздно, уже после Второй Мировой войны. Ее сформулировал в процессе создания теории надежности в 50-е годы в Америке Роберт Луссер — немецкий инженер, конструктор крылатых ракет Fieseler V-1 (Фау-1). Итак: если устройство состоит из определенного количества элементов и мы знаем надежность каждого элемента, то общая надежность системы определяется произведением всех надежностей. Например, если у нас вероятность безотказной работы оси двуколки в течении поездки 0.999, подшипника 0.93 (а их два), а колеса 0.9 (а их тоже два), то получаем 0.999*0.93*0.93*0.9*0.9=0.699868431. Иными словами, наша двуколка надежна всего где-то на 70% и будет ломаться в среднем один раз на 3 поездки… грустно, не так ли?

Такое соединение компонентов называют последовательным. Это не означает, что они соединены в цепочку физически, важно то, что поломка любого из компонентов ведет к выходу из строя всей системы. И не важно, что именно отказало: левый подшипник, правый, или развалилась сама ось. Телега встала, и это главное.

Если у нас в наличии имеются только ненадежные компоненты, можно попытаться сделать из них надежную систему, используя несколько одинаковых компонентов вместо одного. Например, в авиационных поршневых двигателях обязательно используется как минимум два магнето, работающих с отдельными свечами зажигания. Если одно магнето откажет, то двигатель все равно продожит работу. Такое соединение компонентов называется параллельным (опять же, речь идет не о физическом соединении компонентов, а о их роли в системе), а расчитывается оно по простой формуле: 1-(1-p1)*(1-p2)*(1-p3)… где p1, p2, p3 и т.д. есть вероятность безотказной работы каждого из компонентов. Если каждое из магнето имеет надежность 0.99 (1% отказов), то параллельное использование двух магнето будет иметь надежность 1-(1-0.99)*(1-0.99)=0.9999. Имеем 4 девятки после нуля, это уже неплохо. А если поставить три независимых магнето, то получим надежность 0.999999 — один отказ на миллион! Конкретно на эту систему уже можно положиться.

Сложнее, когда надо вычислить надежность работы минимум k элементов из общего количества n. Например, трехмоторный самолет, которому одного мотора не хватает для продолжения полета, но который может спокойно лететь на двух моторах. Для этого надо сложить вероятности наступления всех «хороших» событий, в нашем случае как полета на трех моторах, так и на двух. Вероятность безотказной работы ровно k элементов из n определяется произведением C(n,k)*p^k*(1-p)^(n-k), где C(n,k) — биномиальный коэффициент, который вычисляется как n!/(k!*(n-k)!), p — вероятность безотказной работы одного элемента, n — общее количество элементов и k— минимальное необходимое количество работающих элементов. Полностью формула для системы, состоящей из n одинаковых и равнонадежных элементов, безотказно работают не менее k элементов выглядит так:

k-out-of-n:G

где p(t) — вероятность безотказной работы элемента системы, q(t) = 1 — p(t), ну а биномиальный коэффициент приведен выше.

Итак, пусть надежность работы одного мотора у нас 0.99 (скажем, один отказ на сотню вылетов). Тогда считаем. Считать руками или рисовать красивые красивые картинки с формулами мне было влом, поэтому я просто запустил питон (впрочем, расчет прост и может быть выполнен вручную при необходимости):


>>> from scipy.special import binom
>>> binom(3,2)*pow(0.99,2)*pow((1-0.99),3-2) + binom(3,3)*pow(0.99,3)*pow((1-0.99),0)
0.99970200000000009

Итак, получаем надежность 0.9997 — три фатальных отказа на десять тысяч вылетов, что для боевого самолета уже более-менее приемлемо, тогда как одномоторный самолет с тем же двигателем и одним отказом на сотню вылетов быстро бы забраковали. Кстати, та же самая логика стояла за появлением трехмоторных пассажирских лайнеров для пересечения Атлантики, надежность турбин того времени оставляла желать лучшего. Сейчас же, при нынешней надежности двигателей и способности лайнера лететь на одном двигателе допустимы полеты машин с двумя двигателями.

Вы все еще хотите сесть за штурвал своего самопального пепелаца без предварительного расчета его надежности?

Airplane crash

125 комментариев Закон Луссера

  • dan14444

    Формулы — оно конечно здорово, но поскольку найти достоверные вероятности отказов для каждого элемента — тот ещё геморрой, не говоря о том что элемент обычно не в бинарном состоянии «или сломан или идеально работает»… и от состояния этого элемента зависят другие (например подшипник греется и размягчает изоляцию вблизи)… то реально это всё имеет весьма ограниченную применимость.
    А про резервирование элементов, ессно, отлично знали и в позапрошлом веке…

    Про «неинтуитивность теорвера» же мой любимый пример тут: http://www.membrana.ru/particle/2349
    (и не спешите верить «простым википедийным объяснениям», люди не зря на эту тему математические статьи публикуют…)

    Ну и наконец анекдот «или встречу динозавра, или нет, 50 на 50» — он не анекдот, а великолепная иллюстрация…

    • vashu1

      // мой любимый пример тут: http://www.membrana.ru/particle/2349

      Парадокс конвертов? https://en.wikipedia.org/wiki/Two_envelopes_problem#Simple_resolution

      // не спешите верить «простым википедийным объяснениям

      Обязательно, как только код

      import random

      A = 10
      B = 2 * A

      def look_and_switch(v):
      pass # look
      return True

      def look_and_not_switch(v):
      pass # look
      return False

      def switch(v):
      if v == A:
      return B
      else:
      return A

      s1 = s2 = 0
      for i in range(1000*1000):
      v = random.choice([A, B])
      if look_and_switch(v):
      v = switch(v)
      s1 += v
      v = random.choice([A, B])
      if look_and_not_switch(v):
      v = switch(v)
      s2 += v

      print s1, s2

      покажет что-нибудь кроме
      14989040 15008600

      Вот Parrondo’s paradox серьезная вещь, хоть и довольно банальная. Парадокс Монти Холла имхо куда поучительнее.

      • dan14444

        ага, и откуда например взяли вот это: for i in range(1000*1000) ? ))

        вывод — такие программисты — «это — жулики!» (с) )

        кодь или не кодь, а с тем, что матожидание при обмене есть (2n+n/2)/2 и оно больше n — спорить сложно )

        • olki

          Матожидание принципиально единичного события?
          * Если да, то теорвер таким не занимается
          * Если нет, то Матожидание распределения конечно или нет?
          * * Если да, то меняй конверт iff (тогда и только тогда, когда) в открытом конверте меньше матожидания
          * * Если нет, то ты в любом случае проиграл. В твоем конверте лежат жалкЫе гроши по сравнению с бесконечным матожиданием.

          • dan14444

            не-не-не…

            диапазон — два значения, определяется при вскрытии конверта.
            распределение — два равновероятных значения
            соответственно матожидание для каждого значения в конверте своё, и равно оно (2n+n/2)/2

            а уж какое там максимальное значение — вскрывающему глубоко пофиг,
            как и некое абстрактное матожидание n «до вскрытия, из диапазона и распределения возможных n», сама такая постановка вопроса некорректна ввиду принципиальной недоступности этой инфы.
            если же верхняя граница известна — то понятно, что при приближении к половине оной менять становится невыгодно. но это совсем другая история.

        • Grue

          ага, и откуда например взяли вот это: for i in range(1000*1000) ? ))

          вывод — такие программисты — «это — жулики!» (с) )

          И чего такого жульнического в банальном цикле от 0 до 999999?

          Не, мне взаправду интересно 🙂

        • // матожидание при обмене есть (2n+n/2)/2

          Пардон муа, но в любом обсуждении парадокса конверта выводится что-то вроде

          // То есть в общем виде, если в конверте А лежит сумма С, то статистически ожидаемая сумма в конверте B составит 0,5 х С/2 + 0,5 х 2С = 5/4 С.
          (по вашей ссылке)

          Ну и где тут 5/4 разницы? //14989040 15008600

          Если вам хочется считать единичные события, мы могем.
          for i in range(1000*1000):
          for i in range(1):

          — миллион единичных событий. Принимаю ставки на результат расчета 🙂

          • *
            for i in range(1000*1000):
            for j in range(1):

          • dan14444

            пардоньте, не разглядел, думал это диапазон а не цикл…

            а код считает совсем иное, нужно так:

            цикл
            n — найденное в конверте
            получено при обмене=случайное(n/2, 2n) //считая «случайное» равновероятным выбором из двух
            получено без обмена=n
            суммируем
            крутим до полного удовлетворения

            • Лол, это конечно покажет что вам нужно.

              Только мой код выше прямо эмулирует действия — берем два конверта, даем случайный, всегда свитчим — и, удивительно, не получаем никакой выгоды.

              а этот код
              // получено при обмене=случайное(n/2, 2n) //считая «случайное» равновероятным выбором из двух
              // получено без обмена=n
              эмулирует wishful thinking

              • dan14444

                В том то и дело, что ваш код вносит дополнительные условия, задавая оба значения (и не содержит того самого нарушения симметрии), а мой — точно следует формулировке парадокса ).

                • Да, конечно. А задать значения-то в симуляции как нибудь можно? Рандомно инициализировать например. Или симулировать раздачу, смену и выбор конвертов нельзя, надо считать
                  // получено при обмене=случайное(n/2, 2n)
                  и радоваться? 🙂

                  Вот в моем любимом St. Petersburg paradox в отличии от лабуды с конвертами никакого обмана. Но выигрыша никто не получит, хех.

                  • dan14444

                    да задавайте, чего ж n не задать )
                    Только второе число задавать не надо — вскрывающему оно принципиально недоступно.

                    • olki

                      Если конверт, выбранный игроком и «конверт, в котором генерится первое число из пары чисел» коррелируют, то симметрия нарушена и парадокс сводится к «ты выбрал конверт, а в другом конверте на рубль больше, стОит ли менять?». Ответ — да.

                      Если корелляция отсутствует, то будет 50/50 и «два луя в одну цену». Что мы и наблюдаем в эксперименте.

                    • dan14444

                      Что значит «коррелируют»? Конверты связаны по условию задачи, второе «число» «генерится» в момент фиксации первого наблюдателем.
                      И в соответствующем «эксперименте» мы и наблюдаем близость к матожиданию (2n+n/2)/2

                      Что забавнее, ребята из ссылок в статье утверждают, что стратегия 100% обмена не является наилучшей, и это усугубляет вынос мозга ).

                    • // второе «число» «генерится» в момент фиксации первого наблюдателем.

                      Вся суть. Жаль только в реальных конвертах деньги не генерятся. Тут тумбочка нужна.

                    • Только в условии задачи не сказано что во втором конверте что-то возникает в момент открытия первого, а реальные конверты этим свойством не обладают. Может у вас есть такой конверт? Замечательная вещь.

                    • dan14444

                      Для наблюдателя — именно что не существует. Принципиально недоступная информация. Модель строится в момент фиксации первого числа. Кот Шредингера, если хотите )).

                    • dan14444

                      Еще раз напоминаю — речь идет именно о ПАРАДОКСЕ. О демонстрации ограниченности модели теорвера.

                      И динозавр, и две монетки 33/33/33 — из той же серии ).

                    • // И динозавр

                      Вот уж не знаю где тут ограниченность тервера. Если ничего не знаешь о событии(а блондинка не знает), то назначить 50/50 вполне естественно.

                      Остальное же скорее демонстрирует ограниченность применяющих тервер.

                    • dan14444

                      Ограниченность МОДЕЛИ и практики применения. В положении блондинки люди оказываются гораздо чаще, чем многим представляется. Соответственно и практическая польза, о которой эта статья — обычно преувеличивается заинтересованными и энтузиастами — в ущерб общей эффективности. О чём я и пишу ).

                      А «назначить 50/50 вполне естественно» — это как раз пример классической ошибки или жульничества. Вместо «не знаю» — назвать взятую с потолка цифру, да ещё и якобы обосновать её авторитетом науки ).

                    • // А «назначить 50/50 вполне естественно» — это как раз пример классической ошибки или жульничества.

                      Чтобы вам было легче поставить себя на место блондинки, просто ответьте на вопрос «Смачкуют ли вургли дыблей, да или нет?» Какая вероятность того что ответ «да» верен?

                      // и практическая польза, о которой эта статья — обычно преувеличивается заинтересованными и энтузиастами — в ущерб общей эффективности

                      Если инженеры рассказывают что надежность атомных станций — одна катастрофа во всем мире за 1000 лет, а на практике идет один отказ за 20-30 лет, значит надо переопределять распределения вероятностей.

                      Теория вероятности не может выдать хороший ответ если заложенные распределения — чушь. Но то что люди закладывает неверные распределения это не проблема теории.

                    • // Вместо «не знаю» — назвать взятую с потолка цифру

                      «не знаю» ничего о вероятности одного из двух взаимоисключающих событий === вероятность событий 50/50. Это два представления одного и того же факта.

                      Ничего не знаю -> не получал информации которая могла бы изменить мои оценки -> вероятность равна оценке по умолчанию

                      у нас нет оснований предпочитать одно событие другому -> оценка по умолчанию 50/50

                    • dan14444

                      Я не хочу ставить себя на место блондинки ). А по поводу вопроса — опять повторяю, единственный вменяемый ответ «не знаю». А 50 на 50 — признак безграмотности. Удивляет, что и тут столько народа этого не понимает.

                      И дело, повторюсь, не только в неизвестном распределении, но и в задании вероятностного пространства.
                      О чём замечательно сказал Карлсон Фрекен Бок: «Ты перестала пить коньяк по утрам? отвечай «да» или «нет»»

                      > Если инженеры рассказывают что надежность атомных станций — одна катастрофа во всем мире за 1000 лет, а на практике идет один отказ за 20-30 лет, значит надо переопределять распределения вероятностей.

                      И ещё одно непонимание теорвера и области его применимости… На основании такого наблюдения можно построить встречную модель, оценивающую вероятность адекватности исходной — и не более. Статистическую значимость выборки из 3-4 событий можно обосрать отдельно ).

                      А когда «инженеры рассказывают, что…» и называют такого рода цифры — это то самое дурное применение, о котором я и говорю. Динозавр и прикрытие своих фантазий авторитетом науки.

                      > то что люди закладывает неверные распределения это не проблема теории.

                      угу, это проблема применения этой теории. )

                    • // ужатие вероятностного пространства до пары значений ещё можно принять (с оговорками), но откуда возникает «знание» о распределении?… ))

                      Ужатие вероятностного пространства до пары значений происходит на фазе вопроса, который требует бинарного ответа.

                      «Знание» о распределении возникает из факта что у нас нет никакого знания. А усреднение всех возможных распределений — 50/50. Это просто лучшая оценка распределения в отсутствие какой-либо информации.

                    • dan14444

                      > у нас нет оснований предпочитать одно событие другому -> оценка по умолчанию 50/50

                      ага, вернулись к динозавру )))

                      я не знаю, как ещё объяснить, что это — безграмотность, и что данная модель — не работает, не применима тут…

                    • dan14444

                      > «Знание» о распределении возникает из факта что у нас нет никакого знания. А усреднение всех возможных распределений — 50/50. Это просто лучшая оценка распределения в отсутствие какой-либо информации.

                      Это неверно.

                    • // И дело, повторюсь, не только в неизвестном распределении, но и в задании вероятностного пространства.
                      О чём замечательно сказал Карлсон Фрекен Бок: «Ты перестала пить коньяк по утрам? отвечай «да» или «нет»»

                      У вас есть альтернативы? От того что человеку не нравится произвол в задании вероятностного пространства у него появляются сверхспособности по сравнению с человеком которому произвол нравится?

                      Видя что теория не очень хорошо предсказывает надежность ядерных станций, честный терверщик может попробовать построить модель добавив длинные хвосты в свои распределения и вытащить из модели конкретные рекомендации/ждать опровержения реальностью
                      или сказать — задача на нынешнем уровне не считается.

                      А что может сделать мудрый нелюбитель произвола? Только сказать — задача на нынешнем уровне не считается? Да здравствует мудрость ничегонеделания.

                    • dan14444

                      Вот примерно такими аргументами и пользуются верующие обосновывая свои концепции…
                      (И Союз прибили под девизом «ну надо же что-нибудь делать!» ))

                      Ещё раз, есть методология естествознания, есть бритва Оккама, есть критерии верифицируемости и фальсифицируемости…

                      А заявить про 50/50 — это наплевать на эту методологию и пойти встречать динозавра… если что, 10 раз моргнём и тогда с вероятностью 1-0.5^10=0.999 встретим )))

                    • dan14444

                      P.S. «честный теорверщик», если он ещё и грамотный — прогноз вроде упомянутого «раз в тыщщу лет» делать не будет. Но на заявившего о «не считается» — немедленно находятся конкуренты с принципом Ходжи Насреддина, и радостно обучают ослов…

                    • // наплевать на эту методологию и пойти встречать динозавра… если что, 10 раз моргнём и тогда с вероятностью 1-0.5^10=0.999 встретим

                      А что, это невозможно? Ах да, мы ведь уже ЗНАЕМ что динозавр это физический объект, а они не перезагружаются морганием.

                      С вами говорят о сферическом в вакууме примере незнания, а вы туда тащите свои знания. Такие примеры для того чтобы понимать что значит незнание по мнению теории информации, а не для придирок, противоречащих условиям.

                      // «честный теорверщик», если он ещё и грамотный — прогноз вроде упомянутого «раз в тыщщу лет» делать не будет.

                      Дык уже сделали. И теперь факт несхождения надо объяснить, а заодно решение принять — что с атомными станциями делать.

                      Как будем решение принимать если прогноз не делать? Интуицией? Монетку бросим? Что-то делать надо!

                    • dan14444

                      // наплевать на эту методологию и пойти встречать динозавра… если что, 10 раз моргнём и тогда с вероятностью 1-0.5^10=0.999 встретим
                      /А что, это невозможно? Ах да, мы ведь уже ЗНАЕМ что динозавр это физический объект, а они не перезагружаются морганием.

                      Чой-та? При каждом моргании вас переносит в новый мир с свежими динозаврами… кто сказал нет? 50 на 50! ))

                      //Как будем решение принимать если прогноз не делать? Интуицией? Монетку бросим? Что-то делать надо!

                      Вот-вот, и с Господом Богом так… «Что-то делать надо!» ))
                      Признать неразрешимость данной проблемы на данном уровне и заниматься другими вещами, вместо самообмана. )

            • Hludens

              Занятно как незначительная ошибка в логике может повлиять на результат.
              vashu1 все написал правильно, взял два конверта положил в один денег вдвое больше чем в другой, выдал случайный конверт. Разумеется во втором конверте при этом или вдвое больше или вдвое меньше.
              Вот только при таком раскладе все определено в момент выдачи первого конверта. В среднем сумма денег 1.5Н и как бы вы не брали конверты (кроме случая накопления данных о средней сумме в конверте и последующего выбора второго конверта если в первом денег меньше среднего) вы в результате получите около 1.5*Н*число попыток.
              Любая стратегия, хоть всегда брать первый, хоть всегда брать второй, хоть случайным образом выбирать из первого и второго дадут одинаковый результат.

              А вот если смухлевать и после выбора первой суммы сгенерировать два события Н*2 и Н/2 и случайно выбирать уже из них- результат будет отличаться. Почему?
              Да все просто:
              при честном раскладывании денег у нас наименьшее получаемое число денег Н наибольшее 2Н. т.е. среднее 1.5Н как для первого так и для второго конверта.
              А вот если мы берем два события то диапазон смещается — от Н/2 до 2Н если мы выбираем второй конверт и просто Н если мы оставляем первый. Разумеется среднее значение диапазона [Н/2,2Н] больше Н.

              т.е. неправильно примененная математика подтверждает парадокс.
              В первом случае мы решали задачи с одинаковым набором чисел и поэтому получали результаты которые можно сравнивать.
              А во втором случае две стратегии работают с разным набором чисел потому и дают совершенно разные результаты. Их нельзя сравнивать.

              • Vpotapov1

                /А во втором случае две стратегии работают с разным набором чисел потому и дают совершенно разные результаты. Их нельзя сравнивать./
                более того, раз «правильная» стратегия указывает нам всегда менять конверт, то при формальном применении этой стратегии — получив первый конверт, в него можно и не заглядывать, — ибо зачем, если его все равно обязательно надо сменить? Получается, что мы не пользуемся информацией о сумме в первом конверте никак, и задача вырождается в просто выбор из двух конвертов и бессмысленную смену их после того, как на один показали пальцем.

                • Vpotapov1

                  Забыл подписать: ваш Кэп

                • Hludens

                  Не, ну способ доказательства «поскольку вы не принимаете решений исходя из полученного значения, то оно не важно, и можно просто не смотреть» вполне очевиден и интуитивен, но не объясняет почему математика ясно показывает что нужно открывать второй.
                  Разгадка «парадокса» в неправильном применении математики. Вместо двух связанных случайных действий рассматриваются два не связанных, или вообще упускают из вида первое (первичный выбор «конверта для просмотра»).
                  Из-за этого теряется момент когда мы подменили возможные пределы суммы для второго конверта, а они были определены изначально, до выбора конвертов.

                  • dan14444

                    > они были определены изначально, до выбора конвертов.

                    То, что лежит в конвертах — для наблюдателя принципиально недоступная информация, так что её нет…

                    > Разгадка «парадокса» в неправильном применении математики.

                    Не в «неправильном применении» ИМХО, а в неправильном понимании исходной задачи. И проецировании её на бытовой опыт. Модель соответствует задаче, но сама задача нетривиальна. Возможно также, как нетривиальна логика квантОв. )

                    > Получается, что мы не пользуемся информацией о сумме в первом конверте никак

                    Хе-хе… Почитайте там по ссылочке дальше… Стратегия «всегда менять» — не лучшая (хотя и лучше случайного выбора), там показывают, что если менять используя информацию из вскрытого конверта — получаются и более выгодные стратегии.

                    • Vpotapov1

                      /Хе-хе… Почитайте там по ссылочке дальше… /
                      почитал (суть процитировал ниже, а сразу напишу, что по этому поводу думаю)
                      Дело в том, что не оговорен нижний предел суммы. Ноль не может быть, потому что два умножить на ноль — это не удвоение суммы по сути. И тут возникает два варианта.
                      1. Может быть любая сумма, положим, 10 в минус двенадцатой степени долларов. Тогда, увидев один доллар в конверте, менять его на другой конверт нет никаког смысла (теоретического смысла. Практический смысл в абстрактной задаче не рассматривается).
                      2. Может быть сумма, равная любому натуральному числу, начиная с единицы (к 1 доллара, к примеру). В этом случае мы, увидев в конверте 1 доллар, понимаем, что во втором — 2 доллара. Симметрия теряется, но и задача становится совершенно не интересной.
                      А вот цитата:
                      /Видимый парадокс возник потому, что нельзя избавиться от ощущения, что открытие конверта и наблюдение $10 на самом деле ещё не говорит вам ничего. И поэтому казалось странным, что ожидаемое значение вашего выигрыша в случае смены конверта составляет $12,5, — пояснил Эбботт. — Но мы объясняем этот казус с точки зрения нарушения симметрии. До открытия конвертов ситуация является симметричной, поэтому не имеет значения, будете вы менять потом конверт или нет. Однако после того как вы открываете конверт и используете стратегию Ковера, вы нарушаете симметрию (сразу после открытия конверта А оба конверта уже не равноценны), а затем обмен конвертов позволяет вам получить выгоду в долгосрочном плане (при большом числе заходов)/.

                    • Hludens

                      //для наблюдателя принципиально недоступная информация, так что её нет
                      т.е. вы считаете что когда вы закрываете глаза мир исчезает?
                      Деньги в конверт положены ДО того как вы начнете выбирать. И эти деньги соотносятся как 1:2
                      Если вы действуете согласно некой жесткой стратегии (брать первый, брать второй) то сумма денег в конце эксперимента зависит исключительно от вашего первого выбора, до вскрытия конверта.

                      Парадокс в том что возникает ИЛЛЮЗИЯ того что стратегия всегда менять выигрышная.
                      Собственно математическое моделирование подтверждает это.
                      если моделировать правильно — любая из жестких стратегий дает один и тот же результат.
                      А вот если смоделировать криво и после оглашения некой суммы моделировать случайный выбор двух событий 1/2Н и 2Н то результат начинает резко отличаться от ожидаемого.

                      //Хе-хе… Почитайте там по ссылочке дальше… Стратегия «всегда менять» — не лучшая
                      хе-хе, почитайте внимательней мое первичное сообщение, http://www.popadancev.net/zakon-lussera/comment-page-1/#comment-130811
                      я как раз там и указал что говорю именно о парадоксе и стратегии «всегда менять», а не о очевидном и логичном методе «вычислить среднее и менять только если сумма в конверте меньше чем оно».

                      я накидал простенький код

                      var s1=0;
                      var s2=0;
                      var s3=0;
                      var s4=0;
                      var s5=0;
                      var s6=0;
                      var s7=0;

                      var avg=0;
                      var sum=0;
                      var count=0;

                      for(var i=0; i0.5?[m,2*m]:[2*m,m];

                      s1+=k[0]; //всегда брать первый
                      s2+=k[1]; //всегда брать второй
                      s3+=Math.random()>0.5?k[0]:k[1]; //случайный выбор первого и второго
                      s4+=m; //всегда брать первый для неправильной реализации
                      s5+=Math.random()>0.5?m*2:Math.round(m/2); //неправильная реализация взятия второго конверта

                      s6+=Math.random()>0.5?k[0]*2:Math.round(k[0]/2); //еще одна неправильная реализация взятия всегда второго конверта

                      sum+=k[0]; //стратегия накопления данных о конвертах
                      count++;
                      if(k[0]>sum/count){
                      s7+=k[0];
                      }else{
                      s7+=k[1];
                      sum+=+k[1];
                      count++;
                      }

                      }
                      console.log("всегда брать первый s1=",s1);
                      console.log("всегда брать второй s2=",s2);
                      console.log("случайный выбор первого и второго s3=",s3);
                      console.log("всегда брать первый для неправильной реализации s4=",s4);
                      console.log("неправильная реализация взятия второго конверта s5=",s5);
                      console.log("еще одна неправильная реализация второго конверта s6=",s6);
                      console.log("стратегия накопления данных о конвертах s7=",s7);

                      вот вам результаты его исполнения:

                      всегда брать первый s1= 226201
                      всегда брать второй s2= 225203
                      случайный выбор первого и второго s3= 225953
                      всегда брать первый для неправильной реализации s4= 150468
                      неправильная реализация взятия второго конверта s5= 192082
                      еще одна неправильная реализация второго конверта s6= 282216
                      стратегия накопления данных о конвертах s7= 289913

                      как видите при нормальном моделировании все три первых результата практически одинаковы. диапазон значений для содержимого любого конверта от 100 до 400 (удвоенное 200)
                      Если же мы неверно реализуем код то в случае s4 и s5 мы получаем для первого конверта 100-200 а вот для второго 50-400. Именно поэтому неверно сравнивать результаты испытаний первых трех стратегий с пятой.
                      Шестая стратегия тоже не годится для сравнения. да, ее первый конверт содержит 100-400, но второй конверт уже содержит 50-800!!!
                      У нас внезапно нарисовались лишние деньги… и что с чем мы сравниваем????
                      ну и наконец разумная гибкая стратегия с накоплением данных ожидаемо дала заметно больше денег.

                      Дело в том что когда вы выбираете первый конверт и вскрываете его возможность получить больше или меньше денег в следующем конверте уже предопределена. Если вы изначально взяли конверт с большим числом денег то у вас 100% шанс получить меньше чем в первом и наоборот. То что вы не знаете какой конверт вы выбрали не имеет никакого значения. Что бы вы ни делали вы получите или Н или 2Н.

                    • Vpotapov1

                      Еще немного подумал 🙂
                      это все игры с бесконечностью.
                      При возможности положить в конверт любое число денег, в том числе и какой-нибудь гугол в гугольной степени (знаю, что есть термины и для больших чисел, но неважно), вероятность найти в конверте 1 доллар равна нулю, даже если мы говорим о натуральных числах. Соответственно, это только в реальном мире, где общее число долларов всего-то сотни, ну, пусть тысячи триллионов, есть большая вероятность увидеть один доллар в конверте. Для идеальной задачи, каковой является парадокс, это не катит, так что пусть уважаемый Эббот отдыхает.

                    • dan14444

                      > т.е. вы считаете что когда вы закрываете глаза мир исчезает?

                      Вполне равноправная с обратным гипотеза, весь солипсизм на ней стоит…
                      но я вообще-то о квантаАх, егойных наблюдателях и коллапсе волновой функции )

                      > Если вы действуете согласно некой жесткой стратегии (брать первый, брать второй) то сумма денег в конце эксперимента зависит исключительно от вашего первого выбора, до вскрытия конверта.

                      Ага, а если стратегия принимается после открытия — то результат меняется. В этом и прелесть парадокса.
                      И примитивизированные численные методы и даже статистика тут не помощник, например от эксперименту к эксперименту и диапазон и распределение может злонамеренно меняться, поддерживая только равновероятность n и 2n, а с вычислением среднего будет облом-с 🙂
                      В условиях задачи ничего не сказано сохранении что диапазона, что распределения. 🙂

                      > Еще немного подумал это все игры с бесконечностью.

                      Именно! Натуральными числами значения быть не могут, иначе с нижней границей беда.
                      Значение с одной стороны ограничено нулём (не достигает, но может быть сколь угодно близко), и бесконечно с другой.

                      Возможно ли ввести ограничение сверху? Тут интереснее… придётся обеспечить равновероятность n и 2n, т.е. жёсткую границу ввести не получится. Всегда придётся обеспечивать «двукратный запас над n».

                      Вот это, в частности, я имею ввиду говоря о том что «Модель соответствует задаче, но сама задача нетривиальна.» и проблемах в «проецировании её на бытовой опыт».

                    • dan14444

                      P.S. Самое прелестное в том, что равновероятность n и 2n (со всей сопутствующей бесконечностью) надо соблюдать только для выбирающего.
                      А для формирующего цифры — первую можно задавать произвольно (и менять от эксперимента к эксперименту так, что статистика будет показывать порнографию). Но вот вторую придётся задавать в момент фиксации первой, равновероятным выбором из двух :).

                    • Vpotapov1

                      не знаю, куда попадет ответ (вот где истинный генератор СЧ), но вот что интересно:
                      /Натуральными числами значения быть не могут/
                      вот я как раз подумал, что могут. Да, ситуация становится несимметричной, но не думаете ли Вы, что:
                      «выбор случайного числа от минус бесконечности до плюс бесконечности» и
                      «выбор случайного числа от единицы до плюс бесконечности»
                      это, изящно выражаясь Вашими терминами, статистическая порнография одного порядка

                    • Hludens

                      //Ага, а если стратегия принимается после открытия — то результат меняется. В этом и прелесть парадокса.
                      если стратегия принимается после открытия и ее выбор ЗАВИСИТ от того что вы увидели в конверте — это уже другой случай.
                      Мы разбираем случай описанный в парадоксе, т.е. открыть посмотреть и взять второй поскольку по расчету там должно быть в 1.25 раз больше.
                      Так вот парадокс- это иллюзия. Оценивая смену конвертов отдельно от первоначального выбора, как независимое событие, мы получаем неправильный результат.

                      //например от эксперименту к эксперименту и диапазон и распределение может злонамеренно меняться,
                      не вопрос! при правильно составленном алгоритме все будет штатно работать.
                      Кстати, только что глянул свой код, при публикации какой то бред вылез… видимо нечаянно проглотились пара строк, вот они:

                      for(var i=0; i0.5?[m,2*m]:[2*m,m];

                      так вот, если мы хотим добавить постоянную смену диапазона нужно всего лишь изменить код вот так:


                      for(var i=0; i0.5?[m,2*m]:[2*m,m];

                      Собственно все. в каждом отдельном испытании диапазон значений меняется (если вас смущает число 1000 поставьте больше) но при этом сравнения стратегий в каждом отдельном эксперименте идет по одним и тем же числам.
                      И вуаля! тот же результат!
                      всегда брать первый s1= 135176574
                      всегда брать второй s2= 135476403
                      случайный выбор первого и второго s3= 135389431
                      всегда брать первый для неправильной реализации s4= 90217659
                      неправильная реализация взятия второго конверта s5= 112453793
                      еще одна неправильная реализация второго конверта s6= 168880983
                      стратегия накопления данных о конвертах s7= 163937399

                      Да, стратегия накопления статистики стала несколько менее выгодной, но она по прежнему выгоднее чем жесткие стратегии.

                      Разумеется можно сделать именно злонамеренное изменение, направленное против последней, гибкой стратегии, положив в первые конверты 100 и 200 во второй 75 и 150 и т.д. т.е. добиться вырождения этой стратегии во «всегда брать второй». Но это уже частный случай к парадоксу никакого отношения не имеющий.

                      Ошибка парадокса именно в неправильном применении математики, только к последнему действию, а не ко всем.

                      Аналогичную ошибку делает человек не понимающий парадокс Моунти Холла. Он считает что последняя смена двери ничего не дает, поскольку 50/50 там или коза или автомобиль. Рассматривая свой последний выбор в отрыве от предыдущих, которые сформировали ситуацию, он получает абсолютно неверное значение.

                      //А для формирующего цифры — первую можно задавать произвольно (и менять от эксперимента к эксперименту так, что статистика будет показывать порнографию). Но вот вторую придётся задавать в момент фиксации первой, равновероятным выбором из двух :).
                      какое занятное извращение… Ну хорошо, будь по вашему…

                      var m=Math.round(Math.random()*d+d0);
                      var m2=Math.round(Math.random()>0.5?2*m:m/2);
                      var k=Math.random()>0.5?[m,m2]:[m2,m];

                      и что поменялось?

                      всегда брать первый s1= 101104845
                      всегда брать второй s2= 101416996
                      случайный выбор первого и второго s3= 100941446

                      Вот если зафиксировать выбор и на основе первого выпавшего числа действительно вычислять равновероятно 2н и н/2 тогда да, мы наблюдаем что второй конверт выгоднее. Но это явная ошибка алгоритма, мы начинаем менять деньги во втором конверте после того как посмотрели первый.

      • ElSergV

        Парадокс Монти Холла в невнимательности условий. 2 сразу ошиблись естественно при смене выиграют. 3 варианта 2 ошибочных 1 правильный. но у игрока не 3 ставки, а одна. софистика от математики. (привет кразу)
        Нужен вектор ожиданий, а не нейтральная оцетка. откройте парадокс узника. Для любого узника изначально вероятность 1/2 (казнят его и 1 соседа или 2х соседей), почему-то сказав соседу что казнят другого он поднимает его шансы до 2/3? нифига. изначально неправильное направление мышления. осюда парадокс. отнюдь не гения друг.

    • Grue

      Формулы — оно конечно здорово, но поскольку найти достоверные вероятности отказов для каждого элемента — тот ещё геморрой

      А вы хотели все сразу и нахаляву?

      элемент обычно не в бинарном состоянии «или сломан или идеально работает»… и от состояния этого элемента зависят другие (например подшипник греется и размягчает изоляцию вблизи)

      Если бы я пошел дальше расписывать методы построения блок-схем отказов и способы отслеживания пограничных ситуаций, это уже была бы методичка для ВУЗа приличного объема, а не научно-популярная статья. И утверждение «от состояния этого элемента зависят другие» мягко говоря лукаво, они бывает зависят, а бывает нет, а бывает это все внутри одного агрегата и нам неважно, изоляция от подшипника поплавилась или провода такие ломкие, все равно двигатель ломается с вероятностью 0.95.

      то реально это всё имеет весьма ограниченную применимость

      Все имеет ограниченную применимость, особенно сопромат. Но в некоторых ситуациях без него очень плохо. И, не имея под рукой современные справочники и материалы, лучше поиметь геморрой и трату средств на длительные тесты компонентов на отказ, чем разбиться на сделанном от балды пепелаце.

      А про резервирование элементов, ессно, отлично знали и в позапрошлом веке

      Знать мало, важно обосновать резервирование там, где надо и отсутствие резервирования там, где это приносит больше вреда.

      не анекдот, а великолепная иллюстрация

      …безграмотности.

      люди не зря на эту тему математические статьи публикуют

      Люди херней занимаются из-за того, что игнорируют основы тервера.

      • // А вы хотели все сразу и нахаляву?

        Справедливости ради, когда аборигенам идея покажется неочевидной?

        Двухвинтовые параходы вытеснили одновинтовые на Атлантике в конце 19 века. Несколько предохранительных клапанов на котле делали по крайней мере в начале 19 века. Кто раньше?

        • Grue

          Справедливости ради, когда аборигенам идея покажется неочевидной?

          Двухвинтовые параходы вытеснили одновинтовые на Атлантике в конце 19 века. Несколько предохранительных клапанов на котле делали по крайней мере в начале 19 века. Кто раньше?

          «Опасаясь, чтобы не попасть на каменистые места, бросили с кормы четыре якоря, и ожидали дня.» (c) сами-знаете-кто

        • Vpotapov1

          / Кто раньше?/
          ну как кто? В статье же намекается какбэ — Тот, кто создал существо с двумя глазами

      • dan14444

        //Все имеет ограниченную применимость … Но в некоторых ситуациях без него очень плохо

        К сожалению, в последние полвека обычно «с этим» — куда хуже чем без. Яркий пример — как на таком охмурёже институт проблем управления ВАЗ кушал, смотрим эпопею Березовского )
        Тот случай, когда полуправда хуже лжи, а практика применения — опошляет первоначально здравые идеи.
        Да хоть на недавние «проблемы с экономическим интересом» посмотреть, от глобального потепления и фреонов — до эпопеи свиных/птичьих гриппов и коровьего бешенства…

        Не примите это за наезд на теорвер, у меня в докторской глава на эту тему была, могу прислать как подтверждение некоторого знакомства с темой… )) Но мой опыт показывает, что избыток математики и вообще моделирования без железобетонного обоснуя — обычно плохой признак. Лично меня всегда настораживающий. По крайней мере в биоаналитических статьях и проектах.

        ///не анекдот, а великолепная иллюстрация
        //…безграмотности.

        Кому как, по мне так отличная иллюстрация неочевидности элементарного события. Как в квантах, где подбрасывание двух монеток непринужденно переходит от 25/25/50 в 33/33/33, вырожденность она такая…

        ///люди не зря на эту тему математические статьи публикуют
        //Люди херней занимаются из-за того, что игнорируют основы тервера.

        Всё бывает, и доктора математики херню пишут, и рецензенты её пропускают… Но в таких случаях каг бэ желателен обоснуй с доказательствами, иначе не комильфо.

        • olki

          Что будет, если систему постепенно увеличивать от микро к макро? Скачок 33/33/33 -> 25/50/25, или таки можно получить 30/40/30?

          • dan14444

            В классической парадигме вырождение либо есть либо нет. Возможно ли частичное вырождение — х.з.

            • olki

              Поискал «mesoscopic» two coin toss , но не нашел на первой странице гугла. Вот — как бы по теме: https://physics.stackexchange.com/questions/67346/classical-quantum-coin-toss

              Странно, что статей нет — поидее это тянет на статью в Science/Nature. И опыт поставить не сложно. С помощью соответствующего опыта ЯМР. По аналогии с тем как 15 разложили на множители на «квантовом компьютере» лет 15 назад. У меня-то нынче ямр нет, а ты бы мог сделать статейку…

              • dan14444

                Думаю, «всё придумано до нас» ), про 2 монетки и вырождение я читал то ли в ФЛФ то ли ещё в каком учебнике лет 20 назад…

  • vashu1

    Есть ли у человека вероятностная интуиция, или ещё раз о магии и науке
    https://scholar-vit.livejournal.com/77609.html

    • olki

      Перескажу задачу, чтобы по ссылкам не бегать: Есть 2- и 4-моторный самолёты. Моторы все одинаковые, с вероятностью отказа за время полёта р. Самолёт летит, пока по крайней мере половина моторов работает. На каком безопаснее лететь, в зависимости от значения р?

      Здесь подразумевается, что вероятность отказа двигателя ниже 10%. Иначе это не самолет, в обычном понимании слова. Вывод «четырехмоторный — безопаснее». Замените условие на «при каком шансе штрафного броска 2 из 2 более вероятно, чем 3 из 4». Станет очевидно, что надо рассматривать вариант почти гарантированных попаданий и почти гарантированных промахов.

      В любом случае, ответ не имеет отношения к реальной статистике.

      • olki

        В 1970х цены на нефть выросли. Делать 3х моторные самолеты стало экономически невыгодно. Встала задача доказать, что 2х моторный самолет надежнее 3х и 4х моторного. И это было доказано. (Любой каприз за ваши деньги).

        Задача — повторите вычисления тех специалистов, чтобы доказать, что двухмоторник безопаснее 3х и 4х моторных самолетов. При этом нужно использовать реальные вероятности проблем с двигателями.

      • olki

        Неделя времени на размышления, как доказать, что двухмоторник надежнее трехмоторника (это — правда). Потом напишу ответ.

  • Алекс

    Насчет самолетов — вроде есть существующие ограничения на двухмоторные самолеты, что их маршруты должны пролегать вблизи аэропортов, которые их могут принять в случае отказа двигателя. Для четырехдвигательных ограничения меньше — в частности, через океан могут летать только четырехдвигательные вроде бы.

    • olki

      Years ago, airlines were permitted to fly twin-engine planes no more than sixty minutes from the nearest airfield. Once jets like the 767, 777 and A330 started replacing 747s and DC-10s on the North Atlantic, Boeing and others argued successfully that twin-engine jets were so reliable that it made little sense to restrict them. The costs saved by flying twin engines on long stretches are so compelling that many airlines made the switch on the longest routes.

      В древности были ограничения на дальность полета двухмоторников — летать так, чтобы в пределах часа полета была ВПП [соблюдается для любой точки метрополии США да и для Европы тоже]. Но затем компании доказали, что двухмоторники достаточно надежны и запреты сняли.

      Это — не вся история. Там реально доказали, что двухмоторник безопаснее трехмоторника. [Разгадка — позже]

      • olki

        Не буду томить. Суть такова. У мотора есть три типа повреждений.
        (1) Остановка: 10^-6
        (2) Взрыв: 10^-11
        (2) Отрыв: 10^-14

        Остановка почти всех двигателей не опасна. Двухмоторнику плохо, если вырубается два двигателя (10^-12), трехмоторнику — если вырубается три (10^-18).
        Оторвавшийся двигатель действует как ракета с боеголовкой. Одного отрыва достаточно для уничтожения самолета. Вероятность для двухмоторника: 2*10^-14, трехмоторника: 3*10^-14
        Взорвавшийся двигатель действует как бомба. Для уничтожения машины достаточно взрыва одного двигателя. См «Катастрофа Boeing 747 под Токио».* вероятности 2*10^-11 и 3*10^-11

        Просуммировав варианты катастроф получаем максимальную безопасность на двухмоторниках. Дело усугубляется тем, что очень часто вырубание одного двигателя значит, что из строя выйдут все двигатели. Например, был случай. Рабочие меняли масло. Фильтров от производителя не было. Они поставили похожие фильтры. В полете сначала вырубился один движок, через 10 минут второй, наконец, третий.

        —————
        *Американское — значит отличное. Боинг 747. Фукусима. Ну ты понял.

  • Vpotapov1

    /Вот уж не знаю где тут ограниченность тервера. Если ничего не знаешь о событии(а блондинка не знает), то назначить 50/50 вполне естественно. Остальное же скорее демонстрирует ограниченность применяющих тервер./
    /А «назначить 50/50 вполне естественно» — это как раз пример классической ошибки или жульничества. Вместо «не знаю» — назвать взятую с потолка цифру, да ещё и якобы обосновать её авторитетом науки)./
    «Ничего не знаешь» — это такая вещь в себе. Очень трудно подобрать пример того, что такое «ничего не знаешь». Потому что обстановка, в которой задан вопрос и общие знания — это тоже знания. Приходит на ум пример со «старцем Пигидием». Милонов мог ничего не знать о старце, но уж Невзорова-то он должен был знать…
    А так…
    Вопрос:
    какова вероятность того, что существуют Инопланетные Цивилизации?
    Ответ: 50%, или всякие научные безуспешные программы их поиска таки понизили вероятность существования?
    Вопрос:
    какова вероятность того, что у крокозябры есть кляка?
    Ответ: 50%, мы ничего не знаем ни о крокозябре, ни о кляке

    • dan14444

      Опять динозавр… «50%» — это несравнимо более сильное утверждение чем «не знаю».

      Допустим, ужатие вероятностного пространства до пары значений ещё можно принять (с оговорками), но откуда возникает «знание» о распределении?… ))

  • olki

    если просуммировать правильные ответы навопрос «какова вероятность, что…» которые задают человеку за жизнь, то выяснится, что иногда ответ «повти ноль», иногда «почти пдиница», иногда «0.5». Правильные ответыэти образуют распределение, похожее на гауссиан. С максимумом в 0.5 . Тупо из соображений симметрии.

    Поэтому, если не известно ничего, то правильный ответ «0.5»

    Пример вопроса: «какова вероятность, что китаец может записать слово hui 50 разными иероглифами»?

    Трудно сказать. Значит — 50/50.

    Спойлер: есть плакат «hui, 49 штук». Возможно, укитайцев естьеще один.

    • dan14444

      И ты туда же… а 150 иероглифами (без спойлера) — тоже 50/50? А двадцатью? )

      • olki

        Да, тоже 50/50. Предвосхищая возражение, мол, тогда PROBABILITY(20< COUNT_DISTINCT("hui") Count(«bai»)
        Count(«ba») > Count(«ca»)
        Count(«bai») > Count(«ca»)
        Если дополнительных условий нет вообще, то для первых трех это 1:1.

        Усложним вопрос. Какова вероятность того, что

        Count(«ba») > Count(«bai»)+ Count(«ca»)
        Count(«ca») > Count(«ba»)+ Count(«bai»)
        Count(«bai») > Count(«ba»)+ Count(«ca»)

        Что-то между 1:1 и 1:3.

        Если вопрос `что больше Count(«ba») или Count(«bai»)+ Count(«ca»)` был на китайском выпускном экзамене, то вероятность 1:1 (там на экзаменах сплошная заклепкометрия и наше российское «оценить на пальцах» не котируется).

        Если известно, два из трех вопросов (но неизвестно какие) были на выпускном, то можно сделать вывод, что два слога почти равны по частоте, а третий слог пренебрежимо редок.

        Тогда для утверждения Count(«ca») > Count(«ba»)+ Count(«bai»)

        (1/3) «ca» пренебрежимо редкий, то вероятность равна 0
        (2/3) «b*» пренебрежимо редкий (пусть «bai»), то либо Count(«ca») > Count(«ba»), либо Count(«ca») Count(«ba»)+ Count(«bai») верно с вероятностью 1/3.

        А какова вероятность 1,1*Count(«ca») > Count(«ba»)+ Count(«bai») ? Тоже 1/3.

        Не согласен — прошу улучшить расчет. Только чур у учебник китайского не лезть, grep для подсчета иероглифов не использовать.

        Потом проверим выкладки на частотном анализе греческого и новошумерского.

        • dan14444

          А какое отношение все эти рассуждения имеют к исходной посылке? Ну, кроме того, что для одного значения считать что P(0)=P(50)=0.5 и одновременно P(0)=P(20)=P(50)=P(70)=0.25 как-то неадекватно )))

  • Vpotapov1

    а Росстандарт нам говорит, что, если мы не знаем распределения, то принимаем нормальное

  • Vpotapov1

    почему-то в Вашей дискуссии с vashu1 нет кнопочки «ответить», получается неудобно.
    vashu1 пишет, что делать с предсказанием аварий на атомных станциях.
    Полагаю, что, когда нет еще никакой статистики (ни одной аварии), то любая модель = «не знаю». Когда появится хотя бы одно событие, «не знаю» превращается в «что-то (мало) знаю». Далее появляется статистика и совершенствуется модель. К дискуссии «о динозавре» это не имеет прямого отношения, потому что «нет событий — нет статистики» и теорвер применять нет смысла.
    Вот живет некое племя на склоне вулкана. Они в принципе не знают, что может быть извержение, для них это просто «гора». Для этого племени вопрос «какова вероятность того, что в неопределенном будущем гора может уничтожить племя» не имеет ответа, будь там хоть каждый первый специалистом по теорверу.

    • dan14444

      Матстат — это всё таки не теорвер а его часть, но общий подход верен.

      Либо мы строим модель из фундаментальных соображений (обоснованное вероятностное пространство, распределение и т.д.) на махровом теорвере (см. выше «монетки» обычные и квантовые)
      Либо мы набираем статистику, и строим вероятную модель на её основе, оцениваем её достоверность и т.д…

      А по Росстандарту — динозавры на улице обязательны ).

      Ну и наконец относительно станций… Признав невозможность оценки в исходной форме («сколько раз в тыщщу лет, включая атомную войну и вторжение инопланетян») — никто не мешает рассматривать адекватные модели отдельных элементов и сравнивать риски «при прочих равных». Не забывая о границах применимости. Вот это — адекватно…

  • Vpotapov1

    Хорошо.
    Абстрактная задача.
    Имеется два конверта. Известно, что в каждом из конвертов лежит чек на определенную сумму денег. (ну или две виртуальные кнопки,при нажатии на одну (любую) из них, на счет нажавшему падает некая сумма денег).
    Вопрос. Оптимальная стратегия выбора конверта для получения максимальной суммы денег.
    Ответ. Случайный выбор из двух конвертов (те самые 50%), нет?

    • dan14444

      Разумеется нет.
      Ответ — произвольный выбор. Пофиг по какой стратегии.

      Стратегия равновероятного выбора из этих двух — НЕ даёт преимущества.

      • Vpotapov1

        /Разумеется нет/
        /Ответ — произвольный выбор. Пофиг по какой стратегии./
        Можно еще одну задачу?
        Подбрасывается идеальная монета. Можно делать любую ставку (хоть 1 против 1, хоть 1 против миллиона) как на орла, так и на решку. Мартингейл не предлагать (чтобы не пуститься в обсуждение мартингейла).
        Стратегии:
        1.) ставки 1:1, равновероятный выбор (играющий перед выбором подбрасывает свою идеальную монетку)
        2) ставки 1:10, прочие условия см. п 1
        3.) ставки 1:1, неравновероятный выбор ((играющий перед выбором бросает идеальный 6-гранный кубик, если выпадет единица, то ставит на орла, если остальное — на решку)

        Я полагаю, ни одна из стратегий не дает преимуществ.

        Получается, что преимущества стратегии проявляются в играх типа «ставлю 2 против 1, что при бросании идеального кубика не выпадет единица»

        • Vpotapov1

          И вдогонку. Как только (в моем примере) появился заинтересованный выбор (за выбор любого варианта полагается бабло), волшебным образом исчезает «я не знаю, поэтому и делать ничего не буду», и появляется «надо же что-то делать, хоть монетку кидать, хоть пальцем тыкать, хоть писать две страницы математических выкладок». Другое дело, что писать выкладки вместо тыкания пальцем никакого профита не даст.

          • dan14444

            > за выбор любого варианта полагается бабло), волшебным образом исчезает «я не знаю, поэтому и делать ничего не буду»

            Не-не-не 🙂 «Я не знаю какая стратегия лучше» не равно «я отказываюсь играть» 🙂

            > другое дело, что писать выкладки вместо тыкания пальцем никакого профита не даст.

            Вооот! И можно выбирать всегда первый вариант — он ничем не хуже «случайного равновероятного из двух». И любого другого. Так что никаких 50 на 50.

            • Vpotapov1

              Вам не кажется, что ТЕПЕРЬ особых противоречий в Вашей позиции и позици vashu1 нет?

              • dan14444

                Вы это о динозавре или о чём?

                «50%» и «отсутствие оценки» как были разными вещами, так и остались ))

                • Vpotapov1

                  /«50%» и «отсутствие оценки» как были разными вещами, так и остались ))/
                  я не думаю, что здесь с этим кто-то будет спорить. И у меня вИдение, что никто (включая vashu1 с этим (в такой формулировке) не спорил. Впрочем, я отвечаю, естественно, только за себя.

    • Суть «парадокса» конверта не в том что случайный выбор — лучшая стратегия. Лучшей нет. Но «парадокс» утверждает что лучшая стратегия — слепая замена конвертов.

      • dan14444

        А вы не путайте свою шерсть с государственной.

        «Парадокс конвертов» и этот пример — вещи абсолютно разные, читайте внимательнее ).

        • Ваши «квантовые» конверты вещь конечно забавная, но к унылой реальности никоим боком не относящаяся.

        • Vpotapov1

          /«Парадокс конвертов» и этот пример — вещи абсолютно разные/
          разница в том, что мы имеем знание о содержимом одного из конвертов. Которое в абстрактном случае (а именно, когда предполагается, что сумма может быть сколь угодно большой) не несет никакой информации о возможном содержании второго конверта.
          В конкретном случае, если играющий вскроет конверт и увидит там что-то типа «все деньги мира», то менять он точно ничего не будет, даже если во втором конверте ему предложат «еще немножечко шить»

          • dan14444

            А вот не надо подменять задачу )

            • Vpotapov1

              Подмененная задача (когда в первый конверт, который ОБЯЗАТЕЛЬНО надо подменить, не заглядывают) — по форме — другая, а по сути (полученному результату) такая же.
              Это, примерно, если бы в условии первой задачи было «и плюнуть через левое плечо», я бы предложил плевок исключить, а Вы бы ответили «не надо подменять задачу»

              • dan14444

                по n и 2n в конвертах — cм. соседнюю подветку трёпа. там вы абсолютно верно дошли до «игр с бесконечностью» 🙂

                но не надо путать с банальным обсуждением того, что «равновероятно из двух» неравно «неизвестно» 🙂

          • olki

            Распределение с бесконечным матожиданием? Если да, то твои 100500 денег и даже 2*100500 денег- это гроши по сравнению с бесконечностью. Если нет, то сравнивай с матожиданием.

            dan14444 против вопроса о распределении. Т.е. если тебе пришли «все деньги мира», то в другом конверте все равно с вероятностью 50:50 либо в два раза меньше, либо в два раза больше (это как? «2* все_деньги_мира»?). Запрещено выбирать «всех денег мира мне достаточно». (хотя мне точно хватит прав собственности на всю планету Земля. А dan14444 не хватит.)

            На мой взгляд, vashu1 раскрыл суть. Если создавать конверты по схеме
            * выбираем, какой конверт будет первым, левый или правый.
            * генерируем бабло в первом конверте
            * геренируем коэффициент (*2 или *0.5) в другом конверте
            * создаем бабло и коэффициент*бабло в соответствующих конвертах
            То оптимальная схема выбора — «бери хоть левый, хоть правый. Выбор хоть меняй, хоть не меняй».

            Если есть возможность связать себя с первым или вторым конвертом — связывай себя со вторым.
            Если я правильно понял, то вопрос dan14444 относится к «где та грань, между правый-левый и первый-второй». Вопрос о «всех деньгах мира» он отнесет к категории «это — граничное условие, которое нигде не прописано в условиях задачи».

            • Hludens

              Почти верно 🙂
              Ошибка в деталях.
              Если алгоритм:
              *Создали два конверта, положили в них деньги 1:2
              *теперь выбираем из конвертов первый или посмотрев в первый второй.

              то
              //То оптимальная схема выбора — «бери хоть левый, хоть правый. Выбор хоть меняй, хоть не меняй».

              А вот в описанном вами алгоритме когда сначала берется конверт а потом создаются деньги во втором конверте
              как раз работает фокус со вторым конвертом.

              Причина в том что этот трюк не учитывает первичного выбора.

              Поясню:
              Мы видим некую сумму и пологаем что во втором конверте или вдвое больше или вдвое меньше.
              т.е. видим 100 надеемся что во втором 200, видим 200 надеемся что во втором 400. И именно уту 50% надежду на 400 мы учитываем в нашем вычислении шансов.
              Но! в конвертах изначально лежит некая сумма (например 100 и 200). И если мы взяли конверт с 200 то у нас нет никаких шансов что во втором конверте будет 400.
              т.е. увидем 200 мы надеемся на 400 но абсолютно напрасно.
              И вот как раз эти абсолютно напрасные надежды мы и учитываем в вычислениях. Если вы изначально выбрали конверт с большей суммой смена конверта приведет ТОЛЬКО к потере денег.

              А вот если алгоритм написан с ошибкой и вы вовсе не берете один конверт из существующих, вместо этого вам просто называют случайную сумму, а дальше вы бросаете монетку: орел х2, решка 1/2, то, внезапно, риск оправдан! В случае победы вы получите + 100, а в случае поражения потеряете всего 50!

              Именно в этом и состоит парадокс: беря в руки конверт и глядя на деньги внутри него вы считаете что теперь находитись в ситуации с монеткой, может во втором конверте денег больше, но на самом же деле вы УЖЕ совершили выбор, и, если он был удачен сумма никак не станет больше. Расчеты утверждают что менять конверт нужно, а на самом деле это бессмысленно.

              • Vpotapov1

                што-то вы, господа, подзапутались. Пошел думать)

                • Vpotapov1

                  /А вот в описанном вами алгоритме когда сначала берется конверт а потом создаются деньги во втором конверте/
                  ИМХО ничего не меняется
                  Предположим, вместо конвертов есть два табло, на которых могут высвечиваться некие цифры (для бесконечности табло не хватит, но это уж детали. Пусть будут два бесконеных табло). Ведущий подбрасывает Идеальную Монету, если орел — Число делится пополам, если решка — Число умножается на два.
                  Первый вариант (классический)
                  Ведущему является (именно так, поясню ниже) Число (N), он выводит Число N на Табло № 1, затем подбрасывает Идеальную Монету, и формирует на Табло №2 либо N/2, либо N*2.
                  Далее Игрок выбирает Табло (№1 или №2), Ведущий разворачивает Табло к Игроку, тот видит, что там, принимает решение (менять Табло или оставить выбранное). В зависимости от выбора Игрока, Ведущий выдает сумму, равную тому, что на окончательно выбранном Игроком Табло.

                  Второй вариант (с самозарождением денег в конверте)
                  Ведущему является Число (N), он никаких сигналов ни на одно Табло не посылает (оба Табло остаются пустыми), и просит Игрока выбрать Табло. Игрок выбирает Табло, после чего Ведущий выводит на Табло Число (N) и затем показывает Игроку. Второе Табло остается пустым. Игрок видит на выбранном Табло число N и принимает решение (менять Табло или оставить выбранное). Затем, если Игрок не меняет Табло, Ведущий выдает сумму, равную Числу N, если же Игрок меняет Табло, Ведущий подбрасывает Идеальную Монету и, в зависимости от результата подбрасывания, на (до этого пустом) Табло выводит N/2 или N*2 и выдает Игроку соответстующую сумму.
                  Вот. С ролью Тумбочки прекрасно справляется Табло.
                  Ну Вы же видите, что, во втором варианте перед первоначальным выбором Табло (№1 или №2) Игроком, ведущему нельзя заранее на одно Табло вывести N (да N/2, N и вообще любое), а другое Табло оставить пустым — ну укажет Игрок на пустое Табло, что делать-то? Срочно бросать монетку, прежде чем открыть — так это Вариант 1. Открыть пустое — это существенное изменение сути условий.
                  Дело здесь, повторюсь, в бесконечности (как выше хорошо уточнено, в распределении с бесконечным матожиданием). Число N случайным образом может Ведущему именно что «явиться», никакой Генератор Случайных Чисел не сможет случайным образом выбрать число из бесконечного ряда чисел — памяти не хватит). Аргумент «а вот я вместо ГСЧ взял и выбрал 1» — не катит, примерно на том же основании, что и ответ «0123456789» на вопрос «назовите ряд из 10 случайных чисел», формально не опровергаемый.

                  • Hludens

                    Ох, вчера написал огромный ответ но сбой сервера похоже его проглотил. Попробую воспроизвести.

                    В чем разница между этими двумя вариантами? В первом варианте что то зависит от выбора игрока. Во втором — ничего. Игрок делает выбор но шуллер-крупье подсовывает заранее выбранную карту.

                    Итак сравним эти стратегии. возьмем четырех игроков и одного крупье.
                    У крупье появляется случайное (или строго определенное, тут разницы нет) число.
                    Игрокам 1 и 2 он выводит числа на табло по первому варианту. С игроками 3 и 4 ош действует по второму, т.е. когда они выберут табло выведет им туда это самое число.

                    пусть игроки 1 и 3 всегда выбирают первое из открывшихся чисел, а игроки 2 и 4 посмотрев на эти числа выбирают второе табло.

                    Итак, что мы увидем в конце серии из 1000 игр?

                    для 1 и 2 игрока шансы абсолютно одинаковы. в 50% случаев за выбранным табло скрывается Н, в 25% случаев 2*Н и в оставшихся 25 — 1/2Н т.е. если подсчитать выигрыш он будет 1.125*Н*1000. Причем как у первого так и у второго игрока. Табло равнозначны, поэтому не важно какое из них игрок выберет. и посмотрев на его содержимое он никак не меняет то что на нем написано, а следовательно последовательная замена табло это просто выбор другого равнозначного табло. Правда этим игрокам может повезти или не повезти, если они случайно будут чаще выбирать 2Н их выигрыш окажется несколько больше… так что сумма 1.125Н весьма приблизительна, особенно с малым числом испытаний.

                    Теперь посмотрим на игрока 3. От его выбора ничего не зависит. Ему крупье подпихивает Н так что выигрыш третьего игрока 1000*Н. Ни больше ни меньше.

                    И наконец игрок 4. Может ли он получить Н? Нет, поскольку его ему подкладывают на первое табло. Он всегда получит или 2Н или 1/2Н. 50/50. Шулерство крупье и его стратегия исключили из списка возможных чисел Н. В результате он получит 1.25*Н. И опять таки, от его выбора ничего не зависит.

                    Отсюда вывод: стратегия смотреть в конверт и брать другой работает только если крупье мухлюет.

                    Собственно никакая бесконечность тут не имеет значения. Наши стратегии никак не завязаны на выпавшие значения, поэтому нет никакой разницы что именно выпадает в конвертах, числа от 0 до 1 или от 0 до бесконечности.
                    Величина чисел имеет смысл только при стратегии с накоплением данных, но она то в парадоксе не рассматривается.

                    • Vpotapov1

                      /но сбой сервера похоже его проглотил./ Кэп какбэ намекает, что большие посты перед отправкой хотя бы в буфер Ctrl+C загонять, а лучше в блокнот с сохранением… все знают, но лень-матушка и русское авось… Сам попадался. Спасибо, что не поленились воспроизвести.

                      Теперь по существу.
                      Уточняю: Вы предлагаете следующие стратегии: Игроки 1 и 3 никогда не меняют своего выбора Табло, Игроки 2 и 4 всегда меняют Табло.

                      Если задачу вообще упростить: у Крупье две возможности задать число N: либо N=1, либо N=2, и определяет он, каким будет число N, все тем же бросанием монетки.

                      Легко подсчитать, что при Варианте 1 заранее выводится:
                      Если Крупье выпало N = 1 (с вероятностью 50%), то на Табло 1 выводится 1 рубль, а на Табло 2 выводится равновероятно как уполовинивание, так и удвоение рубля:
                      Табло 1 — 1 рубль, Табло 2 — 0,5 руб
                      Табло 1 — 1 рубль, Табло 2 — 2 руб
                      Если Крупье выпало N = 2(с вероятностью 50%), то равновероятно как уполовинивание, так и удвоение двух рублей:
                      Табло 1 — 2 рубля, Табло 2 — 1 руб
                      Табло 1 — 2 рубля, Табло 2 — 4 рубля

                      Легко подсчитать, что вероятность каждого из четырех раскладов 1/4, иматематическое ожидание суммы при выборе Табло 1 будет (1/4*1+1/4*1+1/4*2+1/4*2) = 1,5 рубля,
                      а при выборе Табло 2 будет (1/4*1/2 +1/4*2+1/4*1+1/4*4)= 1,875 рубля, что ровно на четверть больше, как в классической задаче про конверты и матожидание.

                      Вывод: по Варианту 1 надо всегда выбирать Табло 1 и не менять. Ну, или всегда выбирать Табло 2 и менять))))))))
                      Парадокс как он есть! Даже второго варианта не понадобилось. А Вы говорите, что не в бесконечности тут дело. В ней, родимой.

                    • Vpotapov1

                      Ну перепутал, конечно, Табло 2 надо выбирать и не менять, или Табло 1 выбирать и менять.

                    • Vpotapov1

                      Наверно, надо еще одно бросание монетки ввести: на какое Табло Крупье выводит первоначальные N рублей.
                      Т.е. Крупье бросает монету, результат: N = 1 или N = 2,
                      Затем Крупье второй раз бросает монету, результат: N выводится на Табло 1 или Табло 2
                      Вариант 1
                      Крупье третий раз бросает монету, результат: N/2 или 2N, вместе с N на одном Табло выводит 2N на другом Табло, выбор Табло Игроком и открытие, выбор Игроком смены/несмены Табло

                      Вариант 2
                      Крупье дает выбрать Игроку Табло, потом выводит на него N, показывает Табло Игроку, выбор Игроком смены/несмены, если несмена, то N, если смена, то третье бросание монетки, результат: N или 2N

                      Тогда действительно, по Варианту 1 стратегии Игроков 1 и 2 не отличаются
                      а по Варианту 2 стратегия Игрока 3 (никогда не менять) проигрывает стратегии Игрока 4 (всегда менять)

                      И все-таки, это все относится к конечному выбору N

                      PS Прошу извинить за многабукф

                    • Vpotapov1

                      блиннн.. поправка к Варианту 1
                      вместе с N на одном Табло выводит 2N или N/2 на другом Табло

                    • Vpotapov1

                      четвертую поправку уже не так стыдно)
                      Вариант 2
                      если несмена, то N, если смена, то третье бросание монетки, результат: N/2 или 2N

                    • Hludens

                      нда. вы усложнили объяснения внеся дополнительное случайное действие, и зачем?
                      вот теперь перепрочтите ваши варианты и вы поймете что везде описали вариант 2, т.е. крупье мухлюет и дает игроку первый экран с выпавшим числом Н.
                      т.е. игрок у вас ничего не выбирает, а крупье подсовывает ему выпавший результат как первое значение, а как второе идет 2н или 1/2н что в 1.25 раз больше чем Н.

                      Чем вам не понравились мои примеры с четырьмя игроками и одним крупье? Они полностью описывают всю модель парадокса. игроки 1 и 2 это честная игра, равнозначные конверты, то что подсказывает интуиция. Игроки 3 и 4 это игра с мухлежом, ну или игра на основе ошибочного вычисления, то что подсказывает теорвер в неумелых руках.

                      Вы вмето упрощения задачи запутываете ее невнятными объяснениями. Зачем определять Н? Для серии его можно спокойно принимать хоть как фиксированное число хоть как некий диапазон, это никак не влияет на вычисления.

                      //Легко подсчитать, что вероятность каждого из четырех раскладов 1/4, иматематическое ожидание суммы при выборе Табло 1 будет (1/4*1+1/4*1+1/4*2+1/4*2) = 1,5 рубля,
                      а при выборе Табло 2 будет (1/4*1/2 +1/4*2+1/4*1+1/4*4)= 1,875 рубля, что ровно на четверть больше, как в классической задаче про конверты и матожидание.

                      собственно именно здесь у вас и таится ошибка. вы задали на табло 1 стартовое число а на табло два- вычисленное по коэффициенту.
                      И после этого вы почему то считаете что один игрок всегда будет выбирать табло 1 а другой табло 2! т.е. игрок вообще ничего не выбирает. Он, по вашему, фиксировано берет табло 1! Т.е. задача искусственно вырождается во второй вариант!

                      При нормальной задаче вам нужно добавить бросок монетки Игрока при котором он определяет какое из Табло он откроет первым.
                      т.е. 0.5 что первым будет Табло 1 (с 1.5 рубля) и 0.5 что это будет табло 2 (с 1,875 рубля).
                      т.е. на открытом первым табло будет в среднем (1.5*0.5+1.875*0.5)=1.6875 рубля.
                      игрок номер один берет себе именно эти деньги, а игрок номер два открывает оставшееся табло… И сколько денег мы там видим? внезапно оставшееся табло в 50% случаев это Табло 1, а в оставшихся 50% это Табло 2! т.е. те же (1.5*0.5+1.875*0.5)=1.6875 рубля.

                      Разумеется если вы убираете первоначальный выбор игрока а искусственно пихаете ему в качестве первого числа нечто из одного диапазона а в качестве второго нечто из совсем другого диапазона то очевидная стратегия будет выбирать число с более выгодным диапазоном.

                    • Vpotapov1

                      Прошу извинить, что запутал.
                      /При нормальной задаче вам нужно добавить бросок монетки Игрока при котором он определяет какое из Табло он откроет первым.
                      т.е. 0.5 что первым будет Табло 1 (с 1.5 рубля) и 0.5 что это будет табло 2 (с 1,875 рубля)./

                      Вы правы, вчера я это сам увидел, пост от 19.09.18 время 14:27

                      /Чем вам не понравились мои примеры с четырьмя игроками и одним крупье?/
                      они мне всем понравились.
                      Игроки 3 и 4 демонстрируют, что в Варианте 2 задача стала несимметрична.
                      Более изящный Вариант 2 дан в статье http://faculty.som.yale.edu/barrynalebuff/OtherPersonsEnvelope_JEP1989.pdf
                      сам текст задачи переведен на русский в русскоязычной Вики (задача двух конвертов), внтутри статьи есть задача про Али и Бабу
                      Все, что я вчера навертел (еще раз прошу извинить), написано вот из-за этого:
                      /Собственно никакая бесконечность тут не имеет значения./
                      /Зачем определять Н?/
                      я полагаю, что принципиально, выбираем мы H из бесконечного ряда, или каким-то образом фиксируем. Я решил взять простейший вариант, когда фиксируется минимум — два первых натуральных числа. Я уже потом понял, что задача при этом запутывается вопросами: знают ли Игроки тактику Крупье, знают ли Игроки, что N ограничено двумя значениями и т.п.
                      Собственно, интуитивно понятно, что, если Игрок знает, что N ограничено, и знает верхний предел N, то ему выгодно менять, если число на Табло (или деньги в конверте) близки к N, и невыгодно менять, если далеки.
                      Если же N приниципиально неограничено, то Игроку, увидевшему на Табло стопитьсот тыщ, принять рациональное решение невозможно.
                      Про ограниченность и неограниченность N есть статья:
                      http://rspa.royalsocietypublishing.org/content/early/2009/07/31/rspa.2009.0312
                      забавно, что стратегия «всегда менять» дает выгоду, только она стремится к нулю, если предел не ограничен или Игрок не знает верхнего предела

                    • Hludens

                      //Все, что я вчера навертел (еще раз прошу извинить), написано вот из-за этого:
                      /Собственно никакая бесконечность тут не имеет значения./

                      Ох… и все эти извращения ради того чтобы не писать в условии задачи одну строчку?
                      «Игрок не имеет понятия о порядке суммы в конверте»
                      Этого вполне достаточно.
                      Увидев в конверте 32 рубля вы не можете делать предположения о том, большая из сумм вам досталась или меньшая. Так же как и увидев 423210921032 рубля.
                      Размер чисел в данной задаче не имеет никакого значения.

                      Ну а проблему с накоплением данных при множественных экспериментах решаем заменой порядка проведения опыта, с последовательного на параллельный, т.е. берем 1000 студентов, и одновременно даем каждому по два конверта в которых лежат 32 и 64 рубля. Потом подсчитываем группы (кто менял кто нет) и смотрим сколько в среднем денег у них получилось 🙂

                      Вообще то парадокс описывает единичную ситуацию, а не стратегию на длительный эксперимент, вполне очевидно что при длительном эксперименте наиболее выгодный вариант не «бери второй» а «считай среднее и бери второй если в первом меньше среднего»
                      Так что всякие извращения для борьбы с накоплением знаний заменяются просто вышеуказанной строчкой в условии задачи.

                      Чтобы задача не запутывалась нужно ее ограничивать, а не пытаться охватить все возможные подобные или слегка похожие задачи.

                      Собственно задача двух конвертов элементарная и, правильный ответ на нее: стратегии выбора не существует, оба конверта равнозначны.
                      А парадокс двух конвертов это применение неправильной логики к уже открытому конверту. Пренебрегая тем что выбор уже состоялся, мы учитываем свои ничем не подкрепленные надежды в расчетах, это создает иллюзию того что второй конверт выгоднее.

                      Парадокс именно в том что у задачи с ответом «стратегии нет» образуется математически обоснованная «выгодная стратегия»

                      Стратегии с накоплением данных выходят за рамки парадокса.

                    • dan14444

                      Так, я вернулся и опять поучаствую )

                      > А парадокс двух конвертов это применение неправильной логики к уже открытому конверту. Пренебрегая тем что выбор уже состоялся, мы учитываем
                      > свои ничем не подкрепленные надежды в расчетах, это создает иллюзию того что второй конверт выгоднее.
                      > Парадокс именно в том что у задачи с ответом «стратегии нет» образуется математически обоснованная «выгодная стратегия»

                      Парадокс в том, что «неправильность» логики всё ещё не обоснована. ) После открытия конверта никто не запрещает «пересчитать стратегию заново», и в этот момент «неправильная» логика становится непрошибаемой. И никаких «надежд» там и близко не лежало. )

                      Парадокс на мой взгляд всё же именно в формулировке задачи, хотя она и кажется тривиальной. Ну и отлично иллюстрирует ограниченность наших моделей ). И я вполне допускаю существование реальной системы, в которой именно эта «неправильная» логика будет адекватной моделью. После квантОв и не такое выглядит возможным ).
                      Так что математика и софистика — это хорошо, но эксперимент «властвует над всем». Другое дело, что адекватность эксперимента задаче бывает немногим проще оценить, чем адекватность матмодели…

                    • Hludens

                      //Парадокс в том, что «неправильность» логики всё ещё не обоснована.
                      нет, парадокс именно в том что я описал, а решения парадокса пока нет.
                      Парадокс Зенона в том что Ахилл не может обогнать черепаху, на уровне математики тех времен когда он был высказан он был парадоксом и его решение было совсем не очевидным. Разумеется при применении более подходящего математического аппарата парадокс разваливается.
                      Собственно почти любой парадокс это противоречие нормального, очевидного и математически доказанного.

                      // После открытия конверта никто не запрещает «пересчитать стратегию заново»,
                      Разумеется. Никто не запрещает. Правда просчитывать нужно правильно. Иначе получается парадокс.
                      Ошибка просчета стратегии заключается в том что после вскрытия СЛУЧАЙНО ВЫБРАННОГО конверта мы забываем, что этот выбор был случайным, и исключаем ранее сработавшие вероятности из своих вычислений.

                      Аналогичная логика в парадоксе Монти Холла, там после выбора случайной двери и после вскрытия неслучайной остается две двери, за одной козел за другой автомобиль. Менять свой выбор или нет? Если забыть что предыдущий выбор был СЛУЧАЙНЫЙ, то получается что вероятность 50/50! Менять нет смысла… На самом деле распределение вероятностей совсем иное.

                      // И я вполне допускаю существование реальной системы, в которой именно эта «неправильная» логика будет адекватной моделью.
                      Разумеется! Вот только описываться она будет совсем иначе, точно без заранее предопределенных результатов из которых мы выбираем.

                      //но эксперимент «властвует над всем».
                      Верно. И эксперимент четко показывает что если мы имеем дело с выбором из двух конвертов в которых заранее лежат некие суммы то выигрышной стратегии нет, все равноценны (кроме случая с накоплением статистики).
                      А вот если после нашего выбора пересчитывается содержимое второго конверта, или вместо выбора нам просто предлагается число на основе которого потом вычислится второе число- тогда стратегия «выбирай второй» выгоднее.

                    • Vpotapov1

                      /После открытия конверта никто не запрещает «пересчитать стратегию заново», и в этот момент «неправильная» логика становится непрошибаемой./
                      я правильно понял, что Вы считаете равновероятным нахождение во втором конверте суммы в два раза большей и в два раза меньшей, чем в первом?
                      Причем, «момент» наступает, как правильно только что указали, не при открытии конверта, а еще при выборе одного из двух:
                      1. пока перед нами два конверта, равновероятно нахождение В ОДНОМ (ЛЮБОМ) ИЗ конвертов в суммы в два раза большей и в два раза меньшей, ЧЕМ В ДРУГОМ (КОНКРЕТНО НЕ НАЗВАННОМ)
                      2. как только Игрок укзал пальцем на КОНКРЕТНО ВОТ ЭТОТ, ВЫБРАННЫЙ конверт, появляется выделенный конверт, и высказывание «сумма в ДРУГОМ, НЕВЫБНАННОМ конверте равновероятно либо больше в 2 раза, либо меньше, чем в ВЫБРАННОМ» не является истинным, ибо это другое высказывание, чем в п. 1

              • olki

                Я специально указал, что игрок выбирает между «Левым» и «Правым» конвертом, отмечая либо «Левый» либо «Правый» конверт как «выбранный конверт». Генератор же пользуется понятием «первый» и «второй» конверт. Корреляция между «левым», «первым» и «конвертом, который был выбран игроком» отсутствует.

                Если под «первым» понимать «конверт, выбранный игроком», то, разумеется, симметрия нарушена и надо менять конверт. Если есть корреляция между «выбранный конверт» и «первый» конверт — опять же возникает стратегия.

  • dan14444

    Навеяло всякими «ну надо же что-нибудь делать» и «не знаешь распределения — бери нормальное, оно лучшее», из баша цитатка:

    > злой Йожик:
    демократия — пока лучшее из того политического говна, которое было придумано и опробовано человечеством. недостаток демократии в том, что она неэффективна в слабо-образованных и недоразвитых обществах. пример: иран, пукенистан, оба судана и многие другие
    > Big Macintosh:
    — Больной, вот вам рецепт, получите в аптеке лекарство, будете принимать как тут написано.
    — Доктор, а скоро мне станет лучше?
    — Что за глупый вопрос? Разумеется, лучше вам не станет: у вас на это лекарство аллергия. Но это ваша проблема, потому что это лекарство — лучшее, что придумала медицина за всю свою историю.

    • // недостаток демократии в том, что она неэффективна в слабо-образованных и недоразвитых обществах

      Лол. Что там с образованием и развитием в Англии 17 века было? Может демократия не сводится к вставлению слова «демократический» в название страны?

      Демократия больших бонусов сразу не дает. Она — набор механизмов для разрешения конфликтов в элите при помощи формальных процедур, а не пи..ца революции или произвола смертного автократа. Она уберегает от больших проигрышей, а не дает гигантские выигрыши.

      // у вас на это лекарство аллергия

      Да-да. Уникальный путь, особая цивилизация, лишний ген в хромосоме.

      • dan14444

        Вааще-та приведено было ради второй части, но и насчёт первой можно много разного сказать…

        Ви таки хотите побеседовать за преимущества нынешнего режима в сравнении с Советским и поправкой на 30лет разницы? По уровню социалки, занятости, науки, здравоохранения, образования?…

        Или о том, «кому при демократии жить хорошо», если не брать экономического гегемона после ВВ2, с сателлитами? Ну или греков в Утопии, с 3 рабами на каждого демократа ).
        По факту, пресловутая демократия — одна из форм олигархии, весьма недешёвая в содержании. Считать её «наилучшей» — поостерегусь, а уж для субдоминанта с традиционной ролью врага-пугала для населения доминанта и сателлитов… Не говоря о такой заезженной вещи как «менталитет», который судя по всему таки требует царя-батюшки )).
        И да, уникальный путь и особая цивилизация. Так же, как у японцев, китайцев, англо-саксов, индусов…

      • MegHuk

        А с чего вы взяли, что Англия 17 века была демократишной? Она была аристократическим государством, напомню, пусть и с парламентом. Как и все ее крупные соседи. Демократию с выборами тогда технически позволить себе могли только маленькие вольные торговые города-государства, где все население можно было на площади собрать. И кстати неплохо развивались несмотря на свои скромные размеры и агрессивных соседей.
        Парламентаризм — вот он да, дает те самые механизмы разруливания конфликтов в элитах, но сработают они только тогда, когда у соседа случится кровавый пиздец революции, и элитам станет жутко от подобных перспектив.
        Все верно — толпе невежественных рабов демократию внедрять только портить. Сначала их надо отмыть, накормить и образовать, а с этой задачей справляется неплохо как раз всякие диктатуры развития, а не демократизаторы с авианосцами.

    • Vpotapov1

      Вы усложняете условия «жизненными примерами», а надо бы упрощать и двигаться к абстракции. Вот, опять получился совкосрач — бессмысленный и беспощадный.

      • dan14444

        Написано же — «навеяло» ) Это не пример, это стороннее рассуждение.

        По теме же — утверждение «50 на 50» — более сильное чем «любое». И оно необоснованно и потому отбрасывается. Всё.

  • // Ви таки хотите побеседовать за преимущества нынешнего режима в сравнении с Советским и поправкой на 30лет разницы? По уровню социалки, занятости, науки, здравоохранения, образования?…

    Хм, а от чего проблемы то последние 20 лет? Ах да, вся система навернулась так, что короны^D^D^D партбилеты валялись на мостовой. А отчего? Ах да, мирных формальных способов обновить элиту не было. Удивительно, оказывается если в системе нет способов решать кризисы происходящие раз в пару поколений, то успехи(во многом перерекламированные) на уровне десятилетия-другого уже не так важны — они будут обнулены этим самым кризисом… Можно и дом дешевый построить, а потом через 30 лет проклинать проклятый ураган — ах, если бы не он.

    Ну в союзе хоть элита коллективная была. Зато теперь автократик, старательно обрушивающий альтернативы. Как сдохнет, сразу начнут проклинать проклятых либералов, все разваливших. То что вертикаль десятилетия старательно собиралась в один большой гнилой ствол, рухнувший разом, из поля зрения бедняжек выпадет.

    // уникальный путь и особая цивилизация. Так же, как у японцев, китайцев, англо-саксов, индусов…

    Ну дык Думу распускаем, выборы отменяем, батюшку царя коронуем, адвокатов на биржу труда или как?

    Мы настолько уникальны что сможем совместить абсолютизм с индустриалом? Ах да — уже попробовали, кончилось царственными детишками в колодце. Ну а против уникальности заключающейся в наличии ни за что не отвечающего японского императора я уж не возражаю.

    Есть вполне конкретные изобретения в области науки строительства государства, которые надо прикручивать, приспосабливая к местным условиям. Ну можно конечно рулить в ручном режиме, старательно замыкая все на одного человека. Как все обвалиться, глядишь переоцените стоимость «недешевой» демократии.

    • dan14444

      > Хм, а от чего проблемы то последние 20 лет? Ах да, вся система навернулась так, что короны^D^D^D партбилеты валялись на мостовой. А отчего? Ах да, мирных формальных
      > способов обновить элиту не было. Удивительно, оказывается если в системе нет способов решать кризисы происходящие раз в пару поколений, то успехи(во многом
      > перерекламированные) на уровне десятилетия-другого уже не так важны — они будут обнулены этим самым кризисом… Можно и дом дешевый построить, а потом через 30 лет
      > проклинать проклятый ураган — ах, если бы не он.

      Вах-вах-вах, Союз аднака был экспериментальным образованием, неплохо прошёл 4 смены власти но на пятой — накрылся. Не без помощи, но таки да — с последней сменой власти жопа получилась.
      Но… Те же Штаты намного ли дольше живут по историческим меркам? Не говоря о куче разнообразных государств, в том числе «демократических», не простоявших и одного правления. А империи и монархии жили/живут тысячелетия…
      Так что будем посмотреть )

      > // уникальный путь и особая цивилизация. Так же, как у японцев, китайцев, англо-саксов, индусов…
      > Ну дык Думу распускаем, выборы отменяем, батюшку царя коронуем, адвокатов на биржу труда или как?

      Вы таки хотите от меня расписать адекватное для России государственное устройство прямо здесь и сейчас? Могу предложить навскидку несколько элементов…

      — Наличие центральной фигуры, на которую все надежды, и которая периодически порет особо зарвавшихся бояр. Аналогично нынешней «президентской республике», без увлечения фиговыми листиками… Персонификация государственности, гарант (извиняюсь за выражение). Ибо менталитет-с…

      — Вневыборная передача центральной власти (каковая она и есть, включая Ельцинские времена). Ненаследственная, с кучей ограничений — но с целенаправленной подготовкой преемника и «благословением на царство» предыдущего бугра. Менталитет-с…

      — Некий механизм импичмента, если бугор совсем в маразм впадёт и берега попутает. Всенародно-референдумный, и сложноотменяемый. С завязкой на главную добродетель символа — не попадаться на публичном вранье (как сейчас с пенсионкой, как периодически во внешней политике…). Ну и чтобы особо непопулярные вещи могли принести проблемы (опять пенсионка в пример — сейчас власть срать хотела на подавляющее неодобрение, поскольку революции никто не хочет а иначе никак).

      Да, примерно то же самое обеспечивает двупартийная система «а -ля штаты», но слишком частая грызня за электорат делает её неэффективной, при безумной цене. Достаточно к происходящему сейчас присмотреться — и это в отлаженной-то системе!… Маразм может привести к чему угодно, от обрушения государства до атомной войны. Так что заимствовать «как есть» — не получится, даже если бы сохранились желающие.
      Конкретный механизм — отдельная большая тема.

      — Выборное самоуправление на местах — но уровнем не выше того, где начинается возможность политического сепаратизма.

      > Мы настолько уникальны что сможем совместить абсолютизм с индустриалом?

      Так соседи замечательно совмещали, от Японии начала века, через Франко с Пиночетом и южными корейцами — и до современного Китая. Не чистый абсолютизм ессно, но и никак уж не демократию ).

      > Есть вполне конкретные изобретения в области науки строительства государства, которые надо прикручивать,

      Есть. Но чаще ставят пароходные колёса на трактор — и удивляются результатам.

      > Зато теперь автократик, старательно обрушивающий альтернативы. Как сдохнет, сразу начнут проклинать проклятых либералов, все разваливших. То что вертикаль десятилетия старательно собиралась в один большой гнилой ствол, рухнувший разом, из поля зрения бедняжек выпадет.

      Сам от Пу не в восторге, но Ви таки видите альтернативы? Та гнилая поросль что была до него? Которая сейчас хоть как-то снопом держится? Или «оппозиция», от Ходора с Березовским до Навального? Нафиг-нафиг.
      Единственное, чего хорошего сделал Ельцин — помазал Пу на царствие… Полный был мудак во власти, но власть передал нормально — чего не сделал никто до него с царских времён.
      Иначе было бы хуже.

      > Ну можно конечно рулить в ручном режиме, старательно замыкая все на одного человека. Как все обвалиться, глядишь переоцените стоимость «недешевой» демократии.

      Опять-таки, Ви видите альтернативы? «Недешёвой демократии» «а-ля Рюс» покушали в 90е — всем хватило.
      Одна надежда — Пу таки подготовит и благославит вменяемого преемника, который возможно построит более устойчивую систему.
      Хотя пример Медведа энтузиазма не вызывает…

      • // А империи и монархии жили/живут тысячелетия…

        Конкретнее? Называть Китай многотысячелетней империей, со всеми промежуточными пи,,цами, по сравнению с которыми наша Гражданская война — детская прогулка, мягко говоря неверно. В этом смысле и Киевская Русь — тысячелетие прожила.

        А главное — нам как-то по барабану на опыт доиндустриальных обществ — в индустриальную эру абсолютистские монархии все как одна дали дуба.

        // от Японии начала века, через Франко с Пиночетом и южными корейцами

        От Японии… Что бы было с императором обратившимся в пацифизм? У этих товарищей был богатый опыт по этому делу.

        Про остальных — см определения абсолютизма и авторитаризма.

        // Сам от Пу не в восторге, но Ви таки видите альтернативы?

        В этом то и проблема что все альтернативы старательно уничтожаются. Прежде всего отмена прямых губернаторских выборов — главной кузницы будущих глав государства в нормальных странах.

        Террорист тоже обкладывает жертву бомбами, а потом спрашивает — а какие альтернативы? Тут надо не отвечать на вопрос, а для начала разнести голову террористу. Хотите альтернатив — для начала надо обеспечивать условия для их появления.

        // Одна надежда — Пу таки подготовит и благославит вменяемого преемника

        Дык — одна надежда, вся суть. У юсовцев проблема с преемниками бывает? ИМХО авторитарного правителя учить готовить приемника гнилое дело. Он всю жизнь тренировался использовать обещания преемничества только для разводок.

        // Но чаще ставят пароходные колёса на трактор — и удивляются результатам.

        Прикручивают к волокуше колесо, не касающееся земли и без мотора — и удивляются что только хуже стало.

        Делают суд с около 100% обвинительных, не понимая что фейлы соревновательного суда — хороший механизм для отладки внутренней кухни органов. В результате приходится ориентироваться на случаи вроде https://ru.wikipedia.org/wiki/%D0%A3%D0%B1%D0%B8%D0%B9%D1%81%D1%82%D0%B2%D0%BE_%D0%BD%D0%B0_%C2%AB%D0%96%D0%B4%D0%B0%D0%BD%D0%BE%D0%B2%D1%81%D0%BA%D0%BE%D0%B9%C2%BB Сэкономили на адвокатах, ага — вырастили ментов-бандитов.

        Проводят липовые выборы, не понимая что выборы это не народное управление(какое нах управление по каналу 1 бит в 5 лет), а тест гипножабы. Вон у юсовцев только что реальность напомнила о себе — гипножаба демократов чуть-чуть недотянула, значит подкручивать будут. А у нас, как и в Союзе, нормальных тестов качества гипножабы нет. На комсомольском собрании все изображают пятиминутки ненависти, а дома говорят что в Америке слесарь на лимузине ездит. А потом бац — и многотысячные демонстрации против родной Партии.

        // Наличие центральной фигуры, на которую все надежды, и которая периодически порет особо зарвавшихся бояр.

        Даже Союз эволюционировал к коллективному управлению. Власть Брежнева качественно отличалось от Сталинской.

        Олигархия, в устоявщемся сложном обществе, ИМХО неизбежна. Вопрос как ей напоминать о реальности — хотелось бы что-то понадежнее непредсказуемого автократа.

  • dan14444

    > Конкретнее?

    Китай, Япония, Рим… вполне тысячелетия, и периодические смуты не меняли сути. Да и века той же Британии с Россией тоже не воробей чихнул.
    Насчёт индустриального общества — не вижу препятствий к имперским структурам, разве что масштаб меняется пропорционально средствам связи и транспорту.
    И «абсолютистские монархии» тут ни при чём. Метрополия с символом государственности, провинции, зависимые территории… ничего не напоминает? 🙂

    > В этом то и проблема что все альтернативы старательно уничтожаются.

    Да, это так.
    Но по крайней мере вначале — это ИМХО могло быть оправдано даже «общим благом», РФ была бандитским вертепом. Не до стройки идеальной системы было — надо было хоть какую-то страну собрать. Не до постройки системы противовесов — всё вразнос шло. Потому и рейтинг у Пу по сей день зашкаливает.

    Потом, срока через два — можно было и начать, но с идеальными правителями везде дефицит. А потратив 10 лет на консолидацию власти — отдавать её инстинкты протестуют…

    Возможно, мы наблюдаем выбор в сторону экономических реформ в приоритете перед политическими. Используя капитал любови народной — придавить рубль, прикрывшись внешним врагом и национальной гордостью, прибить остатки Советской социалки… обеспечить конкурентоспособность по азиатскому типу — дешёвой рабочей силой. Под санкции продавить протекционизм… Заставить «элиту» деньги в РФ держать и вообще за рубеж только по диппаспорту…
    (да и внешний враг может внезапно не лубочным оказаться — так что и Осетия и Новороссия могут не худшим выбором оказаться… если НАТО на свой устав не плюнет. )

    А политические — проводить позже, под раздай сметы царя. Тоже стратегия, может и оправданная.
    ЕСЛИ политические реформы будут, или хотя бы передачу власти не сольёт…

    > Прежде всего отмена прямых губернаторских выборов — главной кузницы будущих глав государства в нормальных странах.

    Опять эти «нормальные страны (тм)»… И чем выборные местечковые вожди так уж лучше назначенных?

    > Террорист тоже обкладывает жертву бомбами, а потом спрашивает — а какие альтернативы? Тут надо не отвечать на вопрос, а для начала разнести голову террористу.

    Тот, кто держит за яйца достаточное количество народа — называется не «террорист» а «уважаемый лидер», в таких не стреляют, с такими договариваются.
    Стреляют в тех, у кого хватка ослабла.

    > Дык — одна надежда, вся суть. У юсовцев проблема с преемниками бывает? ИМХО авторитарного правителя учить готовить приемника гнилое дело. Он всю жизнь тренировался использовать обещания преемничества только для разводок.

    Не надо его «учить». Авторитарный правитель таких учителей… Но и воплощением тупого приматизма бугра считать тоже не стоит, свою страну он может любить не меньше прочих, и мозгами обладать тож. Если даже Ельцин смог передать власть — то шансы остаются ).

    > Делают суд с около 100% обвинительных

    Традиция, аднака. И демократия 90х в ней к лучшему не меняла ничего. И не в «экономии на адвокатах» дело. Пока в норме закрытые процессы и амнистии, пока судьи — отдельная и по факту неподотчётная каста — … Эта веточка власти — та ещё вещь в себе…

    > Проводят липовые выборы, не понимая что выборы это не народное управление(какое нах управление по каналу 1 бит в 5 лет), а тест гипножабы. Вон у юсовцев только что реальность напомнила о себе

    Верно. Хотя и не только. Этот 1 бит всё же важен. Его хоть как-то но опасаются, в отличие от. И пар у стада туда уходит. Революции в результате выборов в отработанной системе невозможны, но и терять приоритет у кормушки никто не рвётся, так что некоторая расшивка напряжённости получается.

    По нынешнему напоминанию же — дороговатое напоминание вышло… внешняя политика вразнос, внутри — тоже сюрпризов может быть… Впрочем, бывает хуже.

    В любом случае, к сожалению, попытки прямого заимствования — колесо к волокуше. Система дорогая и сложная в отладке, что вызвало массу удивления в 90е. Оптимисты-реформаторы даже «Незнайку на Луне» явно не читали… Но сделать «как в нормальных странах (тм)» и вот прям сейчас — пытались… суки.

    > Даже Союз эволюционировал к коллективному управлению. Власть Брежнева качественно отличалось от Сталинской.

    Что и стало одной из основных причин коллапса. Символ государства в виде маразматика (и тем более кодлы таковых) до добра не доводит. Символ может быть гадом, но он должен быть жизнеспособным гадом. Или как минимум загадкой. Иначе инстинкты стада не одобрят.

    ИМХО в России на ближайшее столетие персонификация государства обязательна — т.е царь-батюшка, хотя бы комнатный и декоративный. Как и во многих других неплохих социумах (Япония, даже Британия…).
    В других — для удовлетворения хватает гимна, флага и т.д. Пожившие на ЮСовщине знают местное флагодрочество… Есть ли в этом «погрессивность»? — х.з. На мой взгляд, все Луи в одну цену…

    > Олигархия, в устоявщемся сложном обществе, ИМХО неизбежна.

    ИМХО совершенно ниоткуда не следует.

    > Вопрос как ей напоминать о реальности — хотелось бы что-то понадежнее непредсказуемого автократа.

    А никак. Пока не появится кто-то с клещами для яиц — будут играть в свои игры. Классический цикл, чо…

    • // Возможно, мы наблюдаем выбор в сторону экономических реформ в приоритете перед политическими. Используя капитал любови народной — придавить рубль, прикрывшись внешним врагом и национальной гордостью, прибить остатки Советской социалки… обеспечить конкурентоспособность по азиатскому типу — дешёвой рабочей силой. Под санкции продавить протекционизм… Заставить «элиту» деньги в РФ держать и вообще за рубеж только по диппаспорту…

      Если бы режим пилил экономику я бы успокоился.

      Но что имеем? http://investorschool.ru/wp-content/uploads/2016/12/realnyi_VVP_po_godam.jpg

      Во-первых рост начался еще в 98, учитывая инерционность процессов, связывать его с Пу нельзя. Это как раз результат проклятых 90, при всех их минусах — благотворной чистки. Ну и цены на нефть добавим.

      Зато как раз по результатам Пу у нас имеем место прекращение роста в 08. И с тех пор — практически стагнация. И если в нулевые риторика была — удвоим ВВП, то теперь риторика — вокруг враги. Что наводит на мысль что где-то так в 12-13 на экономику решили забить, по отсутствию результатов.

      Удешевление рубля, плоскую шкалу налога, протекционизм еще можно было бы терпеть если бы от этого был толк в росте ВВП — при 10% росте удвоение происходит каждые 7 лет — экспоненциальный рост делает бедняков богаче чем любое перераспределение. Но все это УГ без роста — нет, спасибо. Ничего хорошего в перспективе от текущей политики я лично не вижу.

      • dan14444

        > Если бы режим пилил экономику я бы успокоился.

        Успокойтесь. Он его пилит. Как может. Те же военные расходы давно не растут, гайки тож уже не затягивают, только пояса…

        То, что случился нежданчик и эскалация оказалась нужна некоторым на западе больше чем Пу — ну не всем же быть гениями…
        Яппи вообще вон в своё время на Перл Харбор спровоцировали ).

        Забавно, но при абсолютно тех же репутационных потерях Пу мог всю Украину забрать в сателлита «по просьбе несчастного Янука», выпнув всех несогласных в отделённые западные области… Аки Медвед прошёл по Грузии…

        > Удешевление рубля, плоскую шкалу налога, протекционизм еще можно было бы терпеть

        Забыли про добивание социалки…
        А терпеть придётся по любому :), Незнайка на Луну таки прилетел… вариантов не до хера. Устраивать революцию никому не надо, да и не помогло бы — наоборот. Мечтайте о 10%росте, но радуйтесь тому, что не сильно падает… откачкой от роста нулевых… на фоне санкций животворящих и импортозамещения… (спасибо нефти). Хотя бы радует, что илита нынче вынуждена деньги и детей дома держать.
        В общем, бывает хуже. Смотрим на всех наших правителей с Николая 2 включительно — и видим, что на их фоне Пу не хуже прочих.

    • колесо к волокуше

      https://antontsau.livejournal.com/433050.html
      // Ну и общий взгляд на систему суда присяжных. Бррр. Сидишь, тебе что-то говорят но документы смотреть не дают. Типа «оглашается протокол — блаблабла — огласили, поехали дальше». Понятно откуда ноги растут, в нормальном суде все стороны знакомы с делом и судья тоже, советский суд это формальное изучение доказательств и (как минимум в теории) их реальная оценка по представленному в деле, но тут-то получается что и изучение формальное и оценка не производится. Не могу я оценить вес доказательства если я его только формально увидел. Если не могу вернуться и проверить совпадение с другими. Если не могу запросить уточнения, типа «поясните пожалуйста что мы тут видим на записи» и тп. И при всем при этом вопросы на которые надо отвечать становятся известны на последней стадии — уже после всех выступлений. То есть непонятно на что вообще надо смотреть, не то что изучать подробнее… Теперь, поняв как механизм действует, я бы конечно эти вопросы сам сформулировал исходя из самых начальных бумаг процесса, и обращал бы внимание на то что надо, а так пришлось покрыть записями два десятка листов а4 (причем компутером пользоваться мне не дали, а я пишу так что потом сам с трудом разбираю). Остальные сдохли где-то к середине второго листика, особо задвинутые и один не дописали. И потом по таким вот изучениям со слуха выносится решение о виновности и сроке, не о постановке васеньки в угол… Да, судья потом прокомментировала что если бы она это дело разбирала единолично то признала бы виновными по всем четырем эпизодам. Судья кстати та же самая что вела небезизвестное дело Сутягина и вообще часто слушает дела по всякого рода шпионажам и взяткам, типа за государственные интересы

  • dan14444

    P.S. Есть ещё одна особенность… свойства личности, необходимые для залезания на трон через всю цепочку мелкотравчатых вождей и далее по вектору — совсем не равны свойствам «идеального главы государства». Так что назначение преемника теоретически — эффективнее.
    Детей сейчас назначить (надеюсь), так что некая надежда на объективность подбора — остаётся… Старику может быть важно, как его вспоминать будут — раз уж с династией не получается…

  • Igord

    Даже доказывать ничего не буду и спорить.
    Попробуйте сами понять, где ошибка в статье тутошней.
    Предположим.
    У самолёта два двигателя, они сломаются с вероятностью 50% за один час, каждый из них.
    Какова вероятность отказа двух двигателей за час.

      • Igord

        В реальности по мере усложнения системы, с сопутствующими и временными параметрами будет примерно так.
        Это условность.

        1 двигатель — 50%
        2 двигателя — 30%
        3 двигателя — 26%
        4 двигателя — 24%
        5 двигателей -29%

        • olki

          Студенты МАИ даже объяснят, почему так получается. Взрыв/отрыв одного двигателя почти гарантированно убивает самолет. Так что вероятности складываются. Механики меняли масло и не завинтили одну крышку — вытечет все масло и все движки заглохнут. Число крышек равно числу двигателей.

          В табуне сферических коней в вакууме возникают эффекты, не наблюдаемые в системе с одним конем.

  • olki

    Историки древнего Рима разделяли императоров на «плохих» и «хороших». ИЧСХ «пять хороших императоров» передавали власть переемнику не родственнику. Империя тогда достигла максимального могущества. В том же веке вымерли все патриции.

  • Igord

    Судя по комментариям правильнее было бы… закон Луссера и демократические общества.
    Смешно, типа, мажор потомственный в индустриальном обществе будет квалифицированным чуваком, будет назначать компетентных начальников.

    На Украине, части В. Европы существует олигархическая демократия, плутократия — позволяет только крупным собственникам принимать непосредственное участие в управлении государством. Исполнительная власть подчиняет себе законодательную и судебную. Что есть не демократия, пробуют там суд сделать независимым (многократные поправки к украинской конституции 1996 года не исправили в ней все двусмысленности и противоречия, функционирование правительства не обеспечено чёткой законодательной базой… При этом риск выявления коррупционных сделок крайне невелик и фактически ни одна крупная политическая фигура не была привлечена к ответственности за коррупцию), через МВФ и то не проходит.

    В России система с доминирующей партией, эта партия по сути подчиняется авторитарному лидеру.

    • ElSergV

      Частный случай Закона Паркинсона. Двухуровневая система (т.е. сословная) часто более эффективна…

Leave a Reply to dan14444 Cancel reply

You can use these HTML tags

<a href="" title=""> <abbr title=""> <acronym title=""> <b> <blockquote cite=""> <cite> <code> <del datetime=""> <em> <i> <q cite=""> <s> <strike> <strong>